Re: [R] non-linear regression and root finding

2023-11-07 Thread Ivan Krylov
On Mon, 6 Nov 2023 14:55:39 -0500
J C Nash  wrote:

> However, I'm wondering if this approach is worth writing up, at least
> as a vignette or blog post. It does need a shorter example and some
> explanation of the "why" and some testing perhaps.

Do you mean using this problem as a basis to illustrate ordering
constraints on parameters? Weird constraints do come up every now and
then in regression problems. I could definitely offer my help with at
least some of the text.

> If there's interest, I'll be happy to join in. And my own posting
> suggests how the ordering is enforced by bounding the "delta"
> parameters from below.

I have just tried nlsr::nlxb for a slightly larger dataset shared by
Troels off-list, and it worked great with the delta parameters as you
suggested, thank you!

It's interesting that nlxb and nlsLM give slightly different answers,
differing in 0.5 pK units for pK1 and (pK2-pK1) but not (pK3-pK2). Then
again, they both agree that the standard error for pK1 and (pK2-pK1) is
very large, so perhaps the problem is just very ill-conditioned.

-- 
Best regards,
Ivan

__
R-help@r-project.org mailing list -- To UNSUBSCRIBE and more, see
https://stat.ethz.ch/mailman/listinfo/r-help
PLEASE do read the posting guide http://www.R-project.org/posting-guide.html
and provide commented, minimal, self-contained, reproducible code.


Re: [R] non-linear regression and root finding

2023-11-06 Thread Berwin A Turlach
G'day Troels,

On Tue, 7 Nov 2023 07:14:02 +0100
Troels Ring  wrote:

> Be as it may, I wonder if not your method might work if only we KNOW
> that pK1 is either positive OR negative, in which case we have pK1 =
> -exp(theta1)?

If pK1 can be either negative or positive (or 0 :-) ), and it is just
the ordering that you want to have enforced, then I would try the
parameterisation:

pK1 = pK1
pK2 = pK1 + exp(theta2)
pK3 = pk2 + exp(theta3)

and optimise over pK1, theta2 and theta3.  

As long as you want to know the estimates only.  

Asking for standard errors of the original estimates would open another
can of worms. :-)

Cheers,

Berwin

__
R-help@r-project.org mailing list -- To UNSUBSCRIBE and more, see
https://stat.ethz.ch/mailman/listinfo/r-help
PLEASE do read the posting guide http://www.R-project.org/posting-guide.html
and provide commented, minimal, self-contained, reproducible code.


Re: [R] non-linear regression and root finding

2023-11-06 Thread Troels Ring
Thanks a lot, Berwin. Unfortunately, pK1 may well be negative and as I 
understand the literature it may be poorly defined as such, and also 
seems to be at a boundary, since when lower is set to say rep(-4,3) pK1 
is returned as -4 while pK2 and pK3 are undisturbed. Perhaps the point 
is that pK1 is not carrying any information at the pH around 5. Fair 
enough, I guess. Only, I believe I need stick to including all three pK 
values to be in agreement with the molecular information about HEPES - 
although even this is contentious. Be as it may, I wonder if not your 
method might work if only we KNOW that pK1 is either positive OR 
negative, in which case we have pK1 = -exp(theta1)?


Best wishes
Troels

Den 07-11-2023 kl. 05:10 skrev Berwin A Turlach:

G'day Troels,

On Mon, 6 Nov 2023 20:43:10 +0100
Troels Ring  wrote:


Thanks a lot! This was amazing. I'm not sure I see how the conditiion
pK1 < pK2 < pK3 is enforced?

One way of enforcing such constraints (well, in finite computer
arithemtic only "<=" can be enforced) is to rewrite the parameters as:

pK1 = exp(theta1)   ## only if pK1 > 0
pK2 = pK1 + exp(theta2)
pK3 = pk2 + exp(theta3)

And then use your optimiser to optimise over theta1, theta2 and theta3.

There might be better approaches depending on the specific problem.

Cheers,

Berwin


__
R-help@r-project.org mailing list -- To UNSUBSCRIBE and more, see
https://stat.ethz.ch/mailman/listinfo/r-help
PLEASE do read the posting guide http://www.R-project.org/posting-guide.html
and provide commented, minimal, self-contained, reproducible code.


Re: [R] non-linear regression and root finding

2023-11-06 Thread Berwin A Turlach
G'day Troels,

On Mon, 6 Nov 2023 20:43:10 +0100
Troels Ring  wrote:

> Thanks a lot! This was amazing. I'm not sure I see how the conditiion
> pK1 < pK2 < pK3 is enforced? 

One way of enforcing such constraints (well, in finite computer
arithemtic only "<=" can be enforced) is to rewrite the parameters as:

pK1 = exp(theta1)   ## only if pK1 > 0
pK2 = pK1 + exp(theta2)
pK3 = pk2 + exp(theta3)

And then use your optimiser to optimise over theta1, theta2 and theta3.

There might be better approaches depending on the specific problem.

Cheers,

Berwin

__
R-help@r-project.org mailing list -- To UNSUBSCRIBE and more, see
https://stat.ethz.ch/mailman/listinfo/r-help
PLEASE do read the posting guide http://www.R-project.org/posting-guide.html
and provide commented, minimal, self-contained, reproducible code.


Re: [R] non-linear regression and root finding

2023-11-06 Thread J C Nash

I won't send to list, but just to the two of you, as I don't have
anything to add at this time. However, I'm wondering if this approach
is worth writing up, at least as a vignette or blog post. It does need
a shorter example and some explanation of the "why" and some testing
perhaps.

If there's interest, I'll be happy to join in. And my own posting suggests
how the ordering is enforced by bounding the "delta" parameters from below.

Note that nls() can only handle bounds in the "port" algorithm, and the
man page rather pours cold water on using that algorithm.

Best, JN


On 2023-11-06 14:43, Troels Ring wrote:
Thanks a lot! This was amazing. I'm not sure I see how the conditiion pK1 < pK2 < pK3 is enforced? - it comes from the 
derivation via generalized Henderson-Hasselbalch but perhaps it is not really necessary. Anyway, the use of Vectorize 
did the trick!


Best wishes
Troels

Den 06-11-2023 kl. 19:19 skrev Ivan Krylov:

В Mon, 6 Nov 2023 17:53:49 +0100
Troels Ring  пишет:


Hence I wonder if I could somehow have non linear regression to find
the 3 pK values. Below is HEPESFUNC which delivers charge in the
fluid for known pKs, HEPTOT and SID. Is it possible to have
root-finding in the formula with nls?

Sure. Just reformulate the problem in terms of a function that takes a
vector of predictors (your independent variable SID) and the desired
parameters (pK1, pK2, pK3) as separate arguments and then returns
predicted values of the dependent variable (to compare against pHobs):

kw <- 1e-14 # I'm assuming
pHm <- Vectorize(function(SID, pK1, pK2, pK3)
  -log10(uniroot(HEPESFUNC,c(1e-20,1),tol=1e-20,maxiter=1e4,
 HEPTOT=HEPTOT,SID = SID, pK1=pK1,pK2=pK2,pK3=pK3)$root))

(Yes, Vectorize() doesn't make the function any faster, but I don't see
an easy way to rewrite this function to make it truly vectorised.)

Unfortunately, nls() seems to take a step somewhere where crossprod()
of the Jacobian of the model function cannot be inverted and fails with
the message "Singular gradient". I wish that R could have a more
reliable built-in nonlinear least squares solver. (I could also be
holding it wrong.) Meanwhile, we have excellent CRAN packages nlsr and
minpack.lm:

minpack.lm::nlsLM(
  pHobs ~ pHm(SID, pK1, pK2, pK3),
  data.frame(pHobs = pHobs, SID = SID),
  start = c(pK1 = pK1, pK2 = pK2, pK3 = pK3),
  # the following is also needed to avoid MINPACK failing to fit
  lower = rep(-1, 3), upper = rep(9, 3)
)
# Nonlinear regression model
#   model: pHobs ~ pHm(SID, pK1, pK2, pK3)
#    data: data.frame(pHobs = pHobs, SID = SID)
#    pK1 pK2    pK3
# -1.000   2.966  7.606
#  residual sum-of-squares: 0.001195
#
# Number of iterations to convergence: 15
# Achieved convergence tolerance: 1.49e-08

(Unfortunately, your code seemed to have a lot of non-breakable spaces,
which confuse R's parser and make it harder to copy it.)

I think that your function can also be presented as a degree-5
polynomial in H, so it should also be possible to use polyroot() to
obtain your solutions in a more exact manner.



__
R-help@r-project.org mailing list -- To UNSUBSCRIBE and more, see
https://stat.ethz.ch/mailman/listinfo/r-help
PLEASE do read the posting guide http://www.R-project.org/posting-guide.html
and provide commented, minimal, self-contained, reproducible code.


__
R-help@r-project.org mailing list -- To UNSUBSCRIBE and more, see
https://stat.ethz.ch/mailman/listinfo/r-help
PLEASE do read the posting guide http://www.R-project.org/posting-guide.html
and provide commented, minimal, self-contained, reproducible code.


Re: [R] non-linear regression and root finding

2023-11-06 Thread Troels Ring
Thanks a lot! This was amazing. I'm not sure I see how the conditiion 
pK1 < pK2 < pK3 is enforced? - it comes from the derivation via 
generalized Henderson-Hasselbalch but perhaps it is not really 
necessary. Anyway, the use of Vectorize did the trick!


Best wishes
Troels

Den 06-11-2023 kl. 19:19 skrev Ivan Krylov:

В Mon, 6 Nov 2023 17:53:49 +0100
Troels Ring  пишет:


Hence I wonder if I could somehow have non linear regression to find
the 3 pK values. Below is HEPESFUNC which delivers charge in the
fluid for known pKs, HEPTOT and SID. Is it possible to have
root-finding in the formula with nls?

Sure. Just reformulate the problem in terms of a function that takes a
vector of predictors (your independent variable SID) and the desired
parameters (pK1, pK2, pK3) as separate arguments and then returns
predicted values of the dependent variable (to compare against pHobs):

kw <- 1e-14 # I'm assuming
pHm <- Vectorize(function(SID, pK1, pK2, pK3)
  -log10(uniroot(HEPESFUNC,c(1e-20,1),tol=1e-20,maxiter=1e4,
 HEPTOT=HEPTOT,SID = SID, pK1=pK1,pK2=pK2,pK3=pK3)$root))

(Yes, Vectorize() doesn't make the function any faster, but I don't see
an easy way to rewrite this function to make it truly vectorised.)

Unfortunately, nls() seems to take a step somewhere where crossprod()
of the Jacobian of the model function cannot be inverted and fails with
the message "Singular gradient". I wish that R could have a more
reliable built-in nonlinear least squares solver. (I could also be
holding it wrong.) Meanwhile, we have excellent CRAN packages nlsr and
minpack.lm:

minpack.lm::nlsLM(
  pHobs ~ pHm(SID, pK1, pK2, pK3),
  data.frame(pHobs = pHobs, SID = SID),
  start = c(pK1 = pK1, pK2 = pK2, pK3 = pK3),
  # the following is also needed to avoid MINPACK failing to fit
  lower = rep(-1, 3), upper = rep(9, 3)
)
# Nonlinear regression model
#   model: pHobs ~ pHm(SID, pK1, pK2, pK3)
#data: data.frame(pHobs = pHobs, SID = SID)
#pK1 pK2pK3
# -1.000   2.966  7.606
#  residual sum-of-squares: 0.001195
#
# Number of iterations to convergence: 15
# Achieved convergence tolerance: 1.49e-08

(Unfortunately, your code seemed to have a lot of non-breakable spaces,
which confuse R's parser and make it harder to copy it.)

I think that your function can also be presented as a degree-5
polynomial in H, so it should also be possible to use polyroot() to
obtain your solutions in a more exact manner.



__
R-help@r-project.org mailing list -- To UNSUBSCRIBE and more, see
https://stat.ethz.ch/mailman/listinfo/r-help
PLEASE do read the posting guide http://www.R-project.org/posting-guide.html
and provide commented, minimal, self-contained, reproducible code.


Re: [R] non-linear regression and root finding

2023-11-06 Thread J C Nash

Your script is missing something (in particular kw).

I presume you are trying to estimate the pK values. You may have more success
with package nlsr than nls(). nlsr::nlxb() tries to get the Jacobian of the
model specified by a formula and do so by applying symbolic or automatic
differentiation. The multi expression function would probably not work.
(That is one advantage of nls(), but it has an unstabilized solver and unless
carefully called will use a simple forward derivative approximation.)
There is also nlsr::nlfb() that can use functions for the residuals and
jacobian. Your function is messy, but likely the jacobian can be developed
with a bit of work.

Some of the symbolic derivative features of R can help here. Alternatively,
there are several numerical approximations in nlsr and the vignette "Intro
to nlsr" explains how to use these. Note that simple forward and backward
approximations are generally not worth using, but central approximation is
quite good.

The solver in the nlsr package allows of bounds on the parameters. Since you
want them ordered, you may want to transform

   pK1 < pK2 <- pK3

to  pk1, deltapk2, deltapk3 so pK2 = pk1+deltapk2
and pk3 = pk2 + deltapk3 = pk1 + deltapk2 + deltapk3
and all values bounded below by 0 or possibly some small numbers to
keep the paramters apart.

I won't pretend any of this is trivial. It is VERY easy to make small errors
that still allow for output that is disastrously incorrect. If it is possible
to get a single "formula" as one expression even if spread over multiple lines,
then nlxb() might be able to handle it.


J Nash (maintainer of nlsr and optimx)

On 2023-11-06 11:53, Troels Ring wrote:



HEPESFUNC <-
   function(H,SID,HEPTOT,pK1,pK2,pK3) {
     XX <- (H^3/(10^-pK3*10^-pK2*10^-pK1)+H^2/(10^-pK3*10^-pK2)+H/(10^-pK3))
     IV <- HEPTOT/(1+XX)
     I <- IV*H^3/(10^-pK3*10^-pK2*10^-pK1)
     II <- IV*H^2/(10^-pK3*10^-pK2)
     III <- IV*H/10^-pK3
     H - kw/H + SID + I*2 + II - IV
   }


HEPTOT <- 0.050
SID <- c(-seq(10,1)*1e-4,0,seq(1,10)*1e-4)

pHobs  <- c(4.63,4.68,4.72,4.77,4.83,4.9,4.96,5.04,5.12,5.21,5.3,
     5.39,5.48,5.55,5.63,5.69,5.74,5.8,5.85,5.89,5.93)

pK1 <- -1; pK2 <- 3; pK3 <- 7.55 # literature values
pK3 <- 7.6; pK2 <- 2.96 # values eye-balled to be better

pH <- c()
for (i in 1:length(SID)) {
   pH[i] <- -log10(uniroot(HEPESFUNC,c(1e-20,1),tol=1e-20,maxiter=1e4,
    HEPTOT=HEPTOT,SID = SID[i],
    pK1=pK1,pK2=pK2,pK3=pK3)$root)
}

plot(SID,pH)
points(SID,pHobs,col="red")


__
R-help@r-project.org mailing list -- To UNSUBSCRIBE and more, see
https://stat.ethz.ch/mailman/listinfo/r-help
PLEASE do read the posting guide http://www.R-project.org/posting-guide.html
and provide commented, minimal, self-contained, reproducible code.


Re: [R] non-linear regression and root finding

2023-11-06 Thread Ivan Krylov
В Mon, 6 Nov 2023 17:53:49 +0100
Troels Ring  пишет:

> Hence I wonder if I could somehow have non linear regression to find
> the 3 pK values. Below is HEPESFUNC which delivers charge in the
> fluid for known pKs, HEPTOT and SID. Is it possible to have
> root-finding in the formula with nls?

Sure. Just reformulate the problem in terms of a function that takes a
vector of predictors (your independent variable SID) and the desired
parameters (pK1, pK2, pK3) as separate arguments and then returns
predicted values of the dependent variable (to compare against pHobs):

kw <- 1e-14 # I'm assuming
pHm <- Vectorize(function(SID, pK1, pK2, pK3)
 -log10(uniroot(HEPESFUNC,c(1e-20,1),tol=1e-20,maxiter=1e4,
HEPTOT=HEPTOT,SID = SID, pK1=pK1,pK2=pK2,pK3=pK3)$root))

(Yes, Vectorize() doesn't make the function any faster, but I don't see
an easy way to rewrite this function to make it truly vectorised.)

Unfortunately, nls() seems to take a step somewhere where crossprod()
of the Jacobian of the model function cannot be inverted and fails with
the message "Singular gradient". I wish that R could have a more
reliable built-in nonlinear least squares solver. (I could also be
holding it wrong.) Meanwhile, we have excellent CRAN packages nlsr and
minpack.lm:

minpack.lm::nlsLM(
 pHobs ~ pHm(SID, pK1, pK2, pK3),
 data.frame(pHobs = pHobs, SID = SID),
 start = c(pK1 = pK1, pK2 = pK2, pK3 = pK3),
 # the following is also needed to avoid MINPACK failing to fit
 lower = rep(-1, 3), upper = rep(9, 3)
)
# Nonlinear regression model
#   model: pHobs ~ pHm(SID, pK1, pK2, pK3)
#data: data.frame(pHobs = pHobs, SID = SID)
#pK1 pK2pK3
# -1.000   2.966  7.606
#  residual sum-of-squares: 0.001195
#
# Number of iterations to convergence: 15 
# Achieved convergence tolerance: 1.49e-08

(Unfortunately, your code seemed to have a lot of non-breakable spaces,
which confuse R's parser and make it harder to copy it.)

I think that your function can also be presented as a degree-5
polynomial in H, so it should also be possible to use polyroot() to
obtain your solutions in a more exact manner.

-- 
Best regards,
Ivan

__
R-help@r-project.org mailing list -- To UNSUBSCRIBE and more, see
https://stat.ethz.ch/mailman/listinfo/r-help
PLEASE do read the posting guide http://www.R-project.org/posting-guide.html
and provide commented, minimal, self-contained, reproducible code.


Re: [R] Non-Linear Regression Help

2017-05-10 Thread David Stevens
I have a fair bit of experience with both nls and rating curves. This is 
not a nls() problem, this is a model problem. The power law rating curve 
favored by hydrologists would not apply to your data as it's based on 
the idea that a log-log plot of discharge vs. stage, or state+a in your 
case is a straight line, statistical assumptions notwithstanding. A 
log-log plot of your data,

plot(discharge~stage,data=yourdata,pch=19,log='xy')

clear is not a straight line. The very large discharge at stage=6.53 vs. 
stage=6.32 says one of two things: 1) there is an error in the data 
(perhaps the 2592.05 should be 592.05) or 2) the river channel geometry 
has changed dramatically, as in overtopping its banks or perhaps there's 
a smaller central channel set into a larger flood channel, similar to 
the LA river of the movies. The way forward is 1) recheck your data or 
2) recheck your data and use a two-piece model with one piece for stage 
<= 6.32 and a second piece for stage > 6.32. For this second approach to 
work, you'll need more data than you have given us here.

BTW, nls() should work fine if the model/data combination meet the 
requirements of 1) the model 'fits' the data, 2) the residuals are 
NIID(0,sigma^2), the parameters C, a, and n are identifiable from the 
data (should be if the last point is excluded). As always, you'll need 
good starting values for the parameters (get them from a log-log plot). 
You may find, based on the residuals, that linear regression (lm, glm) 
are most appropriate so that the errors meet the criteria of constant 
variance. If none of this makes sense, buy and study the book

Nonlinear regression analysis: Its applications, D. M. Bates and D. G. 
Watts, Wiley, New York, 1988. ISBN 0471-816434.

The nls() application is the easy part.


Good luck

David Stevens

On 5/4/2017 4:58 PM, Zachary Shadomy wrote:
> I am having some errors come up in my first section of code. I have no
> issue in plotting the points. Is there an easier method for creating a
> non-linear regression using C*(x+a)^n. The .txt file is named
> stage_discharge with the two variables being stage and discharge.
> The data is a relatively small file listed below:
>
> stage discharge
> 6.53 2592.05
> 6.32 559.5782
> 5.96 484.2151
> 4.99 494.7527
> 3.66 456.0778
> 0.51 291.13
>
>
>
>
>
>> power.nls<-nls(stage_dischargee$discharge~C*(stage_discharge$stage+a)^n,
> data=stage_discharge, start=list(C=4, a=0, n=1))
>> C<-coef(power.nls)["C"]
>> a<-coef(power.nls)["a"]
>> n<-coef(power.nls)["n"]
>> plot(stage_discharge$stage, stage_discharge$discharge, pch=17, cex=1.25,
> ylab='Discharge (cfs )', xlab='Stage (ft)', font.lab=2, main='Boone Creek\n
> St age-Discharge Curve')
>> curve(C*(x+a)^n, add=TRUE, col="red")
>   [[alternative HTML version deleted]]
>
> __
> R-help@r-project.org mailing list -- To UNSUBSCRIBE and more, see
> https://stat.ethz.ch/mailman/listinfo/r-help
> PLEASE do read the posting guide http://www.R-project.org/posting-guide.html
> and provide commented, minimal, self-contained, reproducible code.

-- 
David K Stevens, P.E., Ph.D.
Professor and Head, Environmental Engineering
Civil and Environmental Engineering
Utah Water Research Laboratory
8200 Old Main Hill
Logan, UT  84322-8200
435 797 3229 - voice
435 797 1363 - fax
david.stev...@usu.edu



[[alternative HTML version deleted]]

__
R-help@r-project.org mailing list -- To UNSUBSCRIBE and more, see
https://stat.ethz.ch/mailman/listinfo/r-help
PLEASE do read the posting guide http://www.R-project.org/posting-guide.html
and provide commented, minimal, self-contained, reproducible code.


Re: [R] Non-Linear Regression Help

2017-05-05 Thread J C Nash

If you insist on using nls() for anything that you don't understand
extremely well, you will end up with frustration. nls() uses the same
method K F Gauss used (with good understanding of the details) over
200 years ago. The Gauss-Newton approach inside works very well and
efficiently for problems where the assumptions are met, and terribly
most other times. But nls() does have some nice "extras", and rather
than rewrite all the code, we have a wrapnls() function for the codes
in the much more modern nlsr package. It tries (and mostly succeeds) in
getting analytic derivatives in cases like this. Note that nls(), when
you output the diagnostic, tells you that it is having trouble with
the numeric derivative.

I did the following:

1) made a csv file from the data in the posting (Shadomy17.csv)

2) edited the nls() call and added trace and try()

3) ran nlxb() from nlsr. Note that it uses a lot of iterations -- the
problem is quite close to singular. The singular values have NOTHING
to do with the individual parameters. Their display position is just
convenient. Together they show that the ratio of largest / smallest sv
(a measure of the condition number) is very large -- an ill-conditioned
problem. Now we know this -- there's no guessing and hand-waving.

Best, JN

Here's the (rather verbose) output


> library(nlsr)

> shadomy <- read.csv("./Shadomy17.csv")

> power.nls<-try(nls(discharge~C*(stage+a)^n, data=shadomy, start=list(C=4, 
a=0, n=1), trace=TRUE))
7585285 :  4 0 1

> print(power.nls)
[1] "Error in numericDeriv(form[[3L]], names(ind), env) : \n  Missing value or an infinity produced when evaluating the 
model\n"

attr(,"class")
[1] "try-error"
attr(,"condition")


> tmp <- readline("Try a better approach")

> p.nlxb <- nlxb(discharge~C*(stage+a)^n, data=shadomy, start=list(C=4, a=0, 
n=1), trace=TRUE)
formula: discharge ~ C * (stage + a)^n
lower:[1] -Inf -Inf -Inf
upper:[1] Inf Inf Inf
$watch
[1] FALSE

$phi
[1] 1

$lamda
[1] 1e-04

$offset
[1] 100

$laminc
[1] 10

$lamdec
[1] 4

$femax
[1] 1

$jemax
[1] 5000

$rofftest
[1] TRUE

$smallsstest
[1] TRUE

vn:[1] "discharge" "C" "stage" "a" "n"
Finished masks check
datvar:[1] "discharge" "stage"
Data variable  discharge :[1] 2592.05  559.58  484.22  494.75  456.08  291.13
Data variable  stage :[1] 6.53 6.32 5.96 4.99 3.66 0.51
trjfn:
function (prm)
{
if (is.null(names(prm)))
names(prm) <- names(pvec)
localdata <- list2env(as.list(prm), parent = data)
eval(residexpr, envir = localdata)
}


no weights
lower:[1] -Inf -Inf -Inf
upper:[1] Inf Inf Inf
Start:lamda: 1e-04  SS= 7585285  at  C = 4  a = 0  n = 1  1 / 0
lamda: 0.001  SS= Inf  at  C = -843.56  a = 228.63  n = 123.33  2 / 1
lamda: 0.01  SS= Inf  at  C = -515.08  a = 148.03  n = 84.714  3 / 1
lamda: 0.1  SS= 9.0129e+100  at  C = -50.129  a = 37.016  n = 29.648  4 / 1
lamda: 1  SS= 8.5013e+47  at  C = 58.103  a = 30.986  n = 13.954  5 / 1
lamda: 10  SS= 4.2141e+31  at  C = 49.209  a = 45.025  n = 8.0734  6 / 1
lamda: 100  SS= 9.4088e+10  at  C = 17.369  a = 15.101  n = 2.9571  7 / 1
< print(p.nlxb)
nlsr object: x
residual sumsquares =  2468891  on  6 observations
after  31Jacobian and  51 function evaluations
  namecoeff  SE   tstat  pval  gradient
JSingval
C17.2692  1404 0.0123  0.991  -733.5
4112
a  -0.509779 60.67  -0.008403 0.9938   35772   
188.7
n2.44601 31.750.07704 0.9434 -126335  
0.6456

> sink()





On 2017-05-04 06:58 PM, Zachary Shadomy wrote:

I am having some errors come up in my first section of code. I have no
issue in plotting the points. Is there an easier method for creating a
non-linear regression using C*(x+a)^n. The .txt file is named
stage_discharge with the two variables being stage and discharge.
The data is a relatively small file listed below:

stage discharge
6.53 2592.05
6.32 559.5782
5.96 484.2151
4.99 494.7527
3.66 456.0778
0.51 291.13






power.nls<-nls(stage_dischargee$discharge~C*(stage_discharge$stage+a)^n,

data=stage_discharge, start=list(C=4, a=0, n=1))

C<-coef(power.nls)["C"]
a<-coef(power.nls)["a"]
n<-coef(power.nls)["n"]
plot(stage_discharge$stage, stage_discharge$discharge, pch=17, cex=1.25,

ylab='Discharge (cfs )', xlab='Stage (ft)', font.lab=2, main='Boone Creek\n
St age-Discharge Curve')

curve(C*(x+a)^n, add=TRUE, col="red")


[[alternative HTML version deleted]]

__
R-help@r-project.org mailing list -- To UNSUBSCRIBE and more, see
https://stat.ethz.ch/mailman/listinfo/r-help
PLEASE do read the posting guide http://www.R-project.org/posting-guide.html
and provide commented, minimal, self-contained, reproducible code.



__
R-help@r-project.org mailing list -- To UNSUBSCRIBE and more, see
https://stat.ethz.ch/mailman/listinfo/r-help
PLEASE do read the 

Re: [R] Non-Linear Regression Help

2017-05-05 Thread PIKAL Petr
Hi

I am not an expert in nonlinear regression, but your data seems to be rather 
weird. Last five points has almost linear relationship, the first one is 
several times higher. If there is no error in your data, I doubt that you can 
model it by power equation.

Cheers
Petr



> -Original Message-
> From: R-help [mailto:r-help-boun...@r-project.org] On Behalf Of Zachary
> Shadomy
> Sent: Friday, May 5, 2017 12:58 AM
> To: r-help@r-project.org
> Subject: [R] Non-Linear Regression Help
>
> I am having some errors come up in my first section of code. I have no
> issue in plotting the points. Is there an easier method for creating a
> non-linear regression using C*(x+a)^n. The .txt file is named
> stage_discharge with the two variables being stage and discharge.
> The data is a relatively small file listed below:
>
> stage discharge
> 6.53 2592.05
> 6.32 559.5782
> 5.96 484.2151
> 4.99 494.7527
> 3.66 456.0778
> 0.51 291.13
>
>
>
>
>
> > power.nls<-
> nls(stage_dischargee$discharge~C*(stage_discharge$stage+a)^n,
> data=stage_discharge, start=list(C=4, a=0, n=1))
> > C<-coef(power.nls)["C"]
> > a<-coef(power.nls)["a"]
> > n<-coef(power.nls)["n"]
> > plot(stage_discharge$stage, stage_discharge$discharge, pch=17, cex=1.25,
> ylab='Discharge (cfs )', xlab='Stage (ft)', font.lab=2, main='Boone Creek\n
> St age-Discharge Curve')
> > curve(C*(x+a)^n, add=TRUE, col="red")
>
>   [[alternative HTML version deleted]]
>
> __
> R-help@r-project.org mailing list -- To UNSUBSCRIBE and more, see
> https://stat.ethz.ch/mailman/listinfo/r-help
> PLEASE do read the posting guide http://www.R-project.org/posting-
> guide.html
> and provide commented, minimal, self-contained, reproducible code.


Tento e-mail a jakékoliv k němu připojené dokumenty jsou důvěrné a jsou určeny 
pouze jeho adresátům.
Jestliže jste obdržel(a) tento e-mail omylem, informujte laskavě neprodleně 
jeho odesílatele. Obsah tohoto emailu i s přílohami a jeho kopie vymažte ze 
svého systému.
Nejste-li zamýšleným adresátem tohoto emailu, nejste oprávněni tento email 
jakkoliv užívat, rozšiřovat, kopírovat či zveřejňovat.
Odesílatel e-mailu neodpovídá za eventuální škodu způsobenou modifikacemi či 
zpožděním přenosu e-mailu.

V případě, že je tento e-mail součástí obchodního jednání:
- vyhrazuje si odesílatel právo ukončit kdykoliv jednání o uzavření smlouvy, a 
to z jakéhokoliv důvodu i bez uvedení důvodu.
- a obsahuje-li nabídku, je adresát oprávněn nabídku bezodkladně přijmout; 
Odesílatel tohoto e-mailu (nabídky) vylučuje přijetí nabídky ze strany příjemce 
s dodatkem či odchylkou.
- trvá odesílatel na tom, že příslušná smlouva je uzavřena teprve výslovným 
dosažením shody na všech jejích náležitostech.
- odesílatel tohoto emailu informuje, že není oprávněn uzavírat za společnost 
žádné smlouvy s výjimkou případů, kdy k tomu byl písemně zmocněn nebo písemně 
pověřen a takové pověření nebo plná moc byly adresátovi tohoto emailu případně 
osobě, kterou adresát zastupuje, předloženy nebo jejich existence je adresátovi 
či osobě jím zastoupené známá.

This e-mail and any documents attached to it may be confidential and are 
intended only for its intended recipients.
If you received this e-mail by mistake, please immediately inform its sender. 
Delete the contents of this e-mail with all attachments and its copies from 
your system.
If you are not the intended recipient of this e-mail, you are not authorized to 
use, disseminate, copy or disclose this e-mail in any manner.
The sender of this e-mail shall not be liable for any possible damage caused by 
modifications of the e-mail or by delay with transfer of the email.

In case that this e-mail forms part of business dealings:
- the sender reserves the right to end negotiations about entering into a 
contract in any time, for any reason, and without stating any reasoning.
- if the e-mail contains an offer, the recipient is entitled to immediately 
accept such offer; The sender of this e-mail (offer) excludes any acceptance of 
the offer on the part of the recipient containing any amendment or variation.
- the sender insists on that the respective contract is concluded only upon an 
express mutual agreement on all its aspects.
- the sender of this e-mail informs that he/she is not authorized to enter into 
any contracts on behalf of the company except for cases in which he/she is 
expressly authorized to do so in writing, and such authorization or power of 
attorney is submitted to the recipient or the person represented by the 
recipient, or the existence of such authorization is known to the recipient of 
the person represented by the recipient.
__
R-help@r-project.org mailing list -- To UNSUBSCRIBE and more, see
https://stat.ethz.ch/mailman/listinfo/r-help
PLEASE do read the posting guide http://www.R-project.org/posting-guide.html
and provide commented, minimal, self-contained, 

Re: [R] Non linear regression - Von Bertalanffy Growth Function - "singular gradient matrix at initial parameter estimates"

2015-09-08 Thread Xochitl CORMON
Thank you for the tip. Indeed, nlxb in nlmrt works and results are not 
crazy.


I would like however to assess goodness-of-fit (gof) and ultimately to 
compare it with gof from linear regression (fitted with same variables).


Before I used AICc to compare the nls() and lm() fit, however I get now 
an error message concerning the method loglike and its non compatibility 
with nlmrt class object. I guess it is because we use now Marquardt 
method to minimise sum-of square instead of Gauss-Newton? I am right? Or 
this is just an incompatibility coming between AICc function and nlmrt 
objects? Is there an R function to do that?


Best,

Xochitl C.


<>< <>< <>< <><

Xochitl CORMON
+33 (0)3 21 99 56 84

Doctorante en écologie marine et science halieutique
PhD student in marine ecology and fishery science

<>< <>< <>< <><

IFREMER
Centre Manche Mer du Nord
150 quai Gambetta
62200 Boulogne-sur-Mer

<>< <>< <>< <><



Le 19/08/2015 15:11, ProfJCNash a écrit :

Packages nlmrt or minpack.lm use a Marquardt method. minpack.lm won't
proceed if the Jacobian singularity is at the starting point as far as
I'm aware, but nlxb in nlmrt can sometimes get going. It has a policy
that is aggressive in trying to improve the sum of squares, so will use
more effort than nls when both work.

JN

On 15-08-18 12:08 PM, Xochitl CORMON wrote:

Dear all,

I am trying to estimate VBGF parameters K and Linf using non linear
regression and nls(). First I used a classic approach where I estimate
both parameters together as below with "alkdyr" being a subset per year
of my age-length-key database and running in a loop.

vbgf.par <- nls(Lgtcm ~  Linf *(1 - exp(-K * (Age - tzero))), start =
c(K= 0.07, Linf = 177.1), data=alkdyr)

I obtain an estimation of both parameters that are strongly correlated.
Indeed after plotting Linf ~ K and fitting a linear regression I obtain
a function (Linf = a + b*K) with R2= 0.8 and a = 215, b = -763.

In this context, to take into account explicitly correlation between
parameters, I decided to fit a new non linear regression derivate from
VBGF but where Linf is expressed depending on K (I am most interested in
K). To do so, I tried this model:
vbgf.par <- nls(Lgtcm ~  (a + (b*k)) *(1 - exp(-k * (Age - tzero))),
start = c(k= 0.07, a= 215, b=-763), data=alkdyr)

Unfortunately at this point I cannot go further as I get the error
message "singular gradient matrix at initial parameter estimates".

I tried to use alg= plinear (which I am not sure I understand properly
yet). If I give a starting value for a and b only, I have an error
message stating "step factor below minFactor" (even when minFactor is
set to 1000).

Any help will be more than welcome as this is quite urgent

Best,

Xochitl C.






__
R-help@r-project.org mailing list -- To UNSUBSCRIBE and more, see
https://stat.ethz.ch/mailman/listinfo/r-help
PLEASE do read the posting guide http://www.R-project.org/posting-guide.html
and provide commented, minimal, self-contained, reproducible code.


Re: [R] Non linear regression - Von Bertalanffy Growth Function - singular gradient matrix at initial parameter estimates

2015-08-19 Thread ProfJCNash
Packages nlmrt or minpack.lm use a Marquardt method. minpack.lm won't 
proceed if the Jacobian singularity is at the starting point as far as 
I'm aware, but nlxb in nlmrt can sometimes get going. It has a policy 
that is aggressive in trying to improve the sum of squares, so will use 
more effort than nls when both work.


JN

On 15-08-18 12:08 PM, Xochitl CORMON wrote:

Dear all,

I am trying to estimate VBGF parameters K and Linf using non linear
regression and nls(). First I used a classic approach where I estimate
both parameters together as below with alkdyr being a subset per year
of my age-length-key database and running in a loop.

vbgf.par - nls(Lgtcm ~  Linf *(1 - exp(-K * (Age - tzero))), start =
c(K= 0.07, Linf = 177.1), data=alkdyr)

I obtain an estimation of both parameters that are strongly correlated.
Indeed after plotting Linf ~ K and fitting a linear regression I obtain
a function (Linf = a + b*K) with R2= 0.8 and a = 215, b = -763.

In this context, to take into account explicitly correlation between
parameters, I decided to fit a new non linear regression derivate from
VBGF but where Linf is expressed depending on K (I am most interested in
K). To do so, I tried this model:
vbgf.par - nls(Lgtcm ~  (a + (b*k)) *(1 - exp(-k * (Age - tzero))),
start = c(k= 0.07, a= 215, b=-763), data=alkdyr)

Unfortunately at this point I cannot go further as I get the error
message singular gradient matrix at initial parameter estimates.

I tried to use alg= plinear (which I am not sure I understand properly
yet). If I give a starting value for a and b only, I have an error
message stating step factor below minFactor (even when minFactor is
set to 1000).

Any help will be more than welcome as this is quite urgent

Best,

Xochitl C.






__
R-help@r-project.org mailing list -- To UNSUBSCRIBE and more, see
https://stat.ethz.ch/mailman/listinfo/r-help
PLEASE do read the posting guide http://www.R-project.org/posting-guide.html
and provide commented, minimal, self-contained, reproducible code.


Re: [R] Non linear regression - Von Bertalanffy Growth Function - singular gradient matrix at initial parameter estimates

2015-08-18 Thread Bert Gunter
These appear to be primarily statistics/nonlinear optimization issues
that are off topic here, which is about R programming. Post on a
statistics list like stats.stackexchange.com instead.


Cheers,
Bert




Bert Gunter

Data is not information. Information is not knowledge. And knowledge
is certainly not wisdom.
   -- Clifford Stoll


On Tue, Aug 18, 2015 at 9:08 AM, Xochitl CORMON
xochitl.cor...@ifremer.fr wrote:
 Dear all,

 I am trying to estimate VBGF parameters K and Linf using non linear
 regression and nls(). First I used a classic approach where I estimate both
 parameters together as below with alkdyr being a subset per year of my
 age-length-key database and running in a loop.

 vbgf.par - nls(Lgtcm ~  Linf *(1 - exp(-K * (Age - tzero))), start = c(K=
 0.07, Linf = 177.1), data=alkdyr)

 I obtain an estimation of both parameters that are strongly correlated.
 Indeed after plotting Linf ~ K and fitting a linear regression I obtain a
 function (Linf = a + b*K) with R2= 0.8 and a = 215, b = -763.

 In this context, to take into account explicitly correlation between
 parameters, I decided to fit a new non linear regression derivate from VBGF
 but where Linf is expressed depending on K (I am most interested in K). To
 do so, I tried this model:
 vbgf.par - nls(Lgtcm ~  (a + (b*k)) *(1 - exp(-k * (Age - tzero))), start =
 c(k= 0.07, a= 215, b=-763), data=alkdyr)

 Unfortunately at this point I cannot go further as I get the error message
 singular gradient matrix at initial parameter estimates.

 I tried to use alg= plinear (which I am not sure I understand properly yet).
 If I give a starting value for a and b only, I have an error message stating
 step factor below minFactor (even when minFactor is set to 1000).

 Any help will be more than welcome as this is quite urgent

 Best,

 Xochitl C.




 --



 Xochitl CORMON
 +33 (0)3 21 99 56 84

 Doctorante en écologie marine et science halieutique
 PhD student in marine ecology and fishery science



 IFREMER
 Centre Manche Mer du Nord
 150 quai Gambetta
 62200 Boulogne-sur-Mer



 __
 R-help@r-project.org mailing list -- To UNSUBSCRIBE and more, see
 https://stat.ethz.ch/mailman/listinfo/r-help
 PLEASE do read the posting guide http://www.R-project.org/posting-guide.html
 and provide commented, minimal, self-contained, reproducible code.

__
R-help@r-project.org mailing list -- To UNSUBSCRIBE and more, see
https://stat.ethz.ch/mailman/listinfo/r-help
PLEASE do read the posting guide http://www.R-project.org/posting-guide.html
and provide commented, minimal, self-contained, reproducible code.

Re: [R] Non-linear regression analysis in R

2012-12-19 Thread Jeremy Miles
Could you provide the code that you're running, so we can see what
you're trying to do?  Even better would be a repeatable example.

Jeremy

On 19 December 2012 09:42, Yann Labou yann.la...@outlook.com wrote:
 Hey all,

 I'm trying to fit a non-linear model y ~ a * constant ^ b * x ^ c and
 estimates the paramaters a, b and c.

 Using the nls function, I'm getting following error message:

 Error in nlsModel(formula, mf, start, wts) :
   singular gradient matrix at initial parameter estimates

 If I logarithmize the whole equation log(y) ~ log (a) + b * log(constant) +
 c * log(x) and fit the equation with lm, I get only NAs estimates for the
 second term on the right side:

 Coefficients: (1 not defined because of singularities)

 Do you have any hints on how to fit this equation or any alternatives to
 nls?

 Thanks,
 Yann

 __
 R-help@r-project.org mailing list
 https://stat.ethz.ch/mailman/listinfo/r-help
 PLEASE do read the posting guide http://www.R-project.org/posting-guide.html
 and provide commented, minimal, self-contained, reproducible code.

__
R-help@r-project.org mailing list
https://stat.ethz.ch/mailman/listinfo/r-help
PLEASE do read the posting guide http://www.R-project.org/posting-guide.html
and provide commented, minimal, self-contained, reproducible code.


Re: [R] Non-linear regression analysis in R

2012-12-19 Thread Bert Gunter
Jeremy:

Don't be silly. The model is overdetermined -- a and b tradeoff with each
other. e.g. for any solution (a,b), (a/k^m,b+m) for any m is also a
solution, where k = const (assuming I have correctly interpreted the model,
of course).

-- Bert

On Wed, Dec 19, 2012 at 11:59 AM, Jeremy Miles jeremy.mi...@gmail.comwrote:

 Could you provide the code that you're running, so we can see what
 you're trying to do?  Even better would be a repeatable example.

 Jeremy

 On 19 December 2012 09:42, Yann Labou yann.la...@outlook.com wrote:
  Hey all,
 
  I'm trying to fit a non-linear model y ~ a * constant ^ b * x ^ c and
  estimates the paramaters a, b and c.
 
  Using the nls function, I'm getting following error message:
 
  Error in nlsModel(formula, mf, start, wts) :
singular gradient matrix at initial parameter estimates
 
  If I logarithmize the whole equation log(y) ~ log (a) + b *
 log(constant) +
  c * log(x) and fit the equation with lm, I get only NAs estimates for the
  second term on the right side:
 
  Coefficients: (1 not defined because of singularities)
 
  Do you have any hints on how to fit this equation or any alternatives to
  nls?
 
  Thanks,
  Yann
 
  __
  R-help@r-project.org mailing list
  https://stat.ethz.ch/mailman/listinfo/r-help
  PLEASE do read the posting guide
 http://www.R-project.org/posting-guide.html
  and provide commented, minimal, self-contained, reproducible code.

 __
 R-help@r-project.org mailing list
 https://stat.ethz.ch/mailman/listinfo/r-help
 PLEASE do read the posting guide
 http://www.R-project.org/posting-guide.html
 and provide commented, minimal, self-contained, reproducible code.




-- 

Bert Gunter
Genentech Nonclinical Biostatistics

Internal Contact Info:
Phone: 467-7374
Website:
http://pharmadevelopment.roche.com/index/pdb/pdb-functional-groups/pdb-biostatistics/pdb-ncb-home.htm

[[alternative HTML version deleted]]

__
R-help@r-project.org mailing list
https://stat.ethz.ch/mailman/listinfo/r-help
PLEASE do read the posting guide http://www.R-project.org/posting-guide.html
and provide commented, minimal, self-contained, reproducible code.


Re: [R] Non linear regression with complex equation

2012-02-28 Thread jeff_hawkes
Apologies for the phrasing of the question.
I've sorted the problem (thanks Bert Gunter) by using the curly brackets {}
as below (using a simplified version of my real model). I hope this helps
someone else!

Jeff
---
 data
  Alpha ip   X
1 0.7106967  0.3616727 0.006027879
2 2.1678517  5.0615917 0.084359861
3 4.4066250 11.2282945 0.187138242
4 9.8495694 18.0534974 0.300891624
527.7247098 29.2064434 0.486774057
670.6931430 35.3946092 0.589910153
7   133.1240255 46.0347288 0.767245480
8   214.7851844 49.3811149 0.823018582
9   359.5511036 58.5069583 0.975115972
10  748.1840127 57.3744477 0.956240795
11 2129.9844080 60.000 1.0
 c-1.83e-9
 cFe=c
 model-nls({Fe1-cFe/(Alpha+1+k*c)
+ X~Alpha/(Alpha+1+k*c/(1+k*Fe1))},start=list(k=1e10))
 summary(model)

Formula: X ~ Alpha/(Alpha + 1 + k * c/(1 + k * Fe1))

Parameters:
   Estimate Std. Error t value Pr(|t|)
k 3.491e+10  7.190e+09   4.856 0.000665 ***
---
Signif. codes:  0 ‘***’ 0.001 ‘**’ 0.01 ‘*’ 0.05 ‘.’ 0.1 ‘ ’ 1 

Residual standard error: 0.05589 on 10 degrees of freedom

Number of iterations to convergence: 8 
Achieved convergence tolerance: 2.393e-06 


--
View this message in context: 
http://r.789695.n4.nabble.com/Non-linear-regression-with-complex-equation-tp4425942p4427617.html
Sent from the R help mailing list archive at Nabble.com.

__
R-help@r-project.org mailing list
https://stat.ethz.ch/mailman/listinfo/r-help
PLEASE do read the posting guide http://www.R-project.org/posting-guide.html
and provide commented, minimal, self-contained, reproducible code.


Re: [R] Non linear regression with complex equation

2012-02-27 Thread Rolf Turner


Your question is (completely) ill-posed.  What is your actual
model?  What you have said makes no sense at all as it stands.
(Minimal self-contained example .)

cheers,

Rolf Turner

On 28/02/12 09:25, jeff_hawkes wrote:

Hi all,
Is it possible to model a function where the unknown parameter appears both
in the fitted equation AND in the determination of other parameters?  E.g.

y = a^2 + b/2 + k

where a = 2/k  and b = k^2

and the model needs to determine k?  I know this is a very simple equation
(its just an example), the one I am modelling is much more complicated!

k appears in the equation which the n.l.r model fits, but it also affects
other parameters in the equation.  Please let me know if you know a way of
achieving this.  I realise it is possible to set up a loop where the
modelled value for k is fed back in to a and b, and the model is run again -
but it seems like there should be a more elegant way within one run of the
model.


__
R-help@r-project.org mailing list
https://stat.ethz.ch/mailman/listinfo/r-help
PLEASE do read the posting guide http://www.R-project.org/posting-guide.html
and provide commented, minimal, self-contained, reproducible code.


Re: [R] Non-linear Regression best-fit line

2011-06-18 Thread Dennis Murphy
Hi:

Perhaps the self-starting functions may be helpful. See ?selfStart.
There are self-starting functions for both the logistic and Gompertz
models (SSlogis and SSgompertz, respectively). Go through the examples
to see how they work.

HTH,
Dennis

On Fri, Jun 17, 2011 at 2:14 PM, Sean Bignami bignam...@gmail.com wrote:
 I am trying to fit a curve to a cumulative mortality curve (logistic) where y 
 is the cumulative proportion of mortalities, and t is the time in hours (see 
 below). Asym. at 0 and 1
 y
  [1] 0. 0.04853859 0.08303777 0.15201970 0.40995074 0.46444992 
 0.62862069 0.95885057 1.
 [10] 1. 1.
 t
  [1]  0 13 20 24 37 42 48 61 72 86 90

 I tried to find starting values for a Gompertz non-linear regression by 
 converting the equation (y~1*exp(-beta*exp(-gamma*t)) to a linear form per 
 Dalgaard Introductory Statistics with R pg.279-280. But got this Error 
 message:

 lm(log(0-log(y))~t)
 Error in lm.fit(x, y, offset = offset, singular.ok = singular.ok, ...) :
  NA/NaN/Inf in foreign function call (arg 4)

 I tried to change all by 0 and 1 values to non-zero and non-one values (yy 
 and tt below), and was able to get starting estimates.

 yy-c(0.1,0.04853859, 0.08303777, 0.15201970, 0.40995074, 
 0.46444992, 0.62862069, 0.95885057, 
 0.99,0.,0.)
 tt-c(0.01,13,20,24,37,42,48,61,72,86,90)

 lm(log(0-log(yy))~tt)

 Call:
 lm(formula = log(0 - log(yy)) ~ tt)

 Coefficients:
 (Intercept)           tt
     9.5029      -0.3681

 However, when I plug those values into the nls function, I get an error 
 message about the getInitial method

 nlsout-nls(y~1*exp(-beta*exp(-gamma*t),start=c(beta=exp(9.5),gamma=.368)));summary(nlsout)
 Error in getInitial.default(func, data, mCall = as.list(match.call(func,  :
  no 'getInitial' method found for function objects

 Can anyone help clarify how I can find the parameters for a best-fit curve 
 for this data? Thanks!!

 Sean
        [[alternative HTML version deleted]]

 __
 R-help@r-project.org mailing list
 https://stat.ethz.ch/mailman/listinfo/r-help
 PLEASE do read the posting guide http://www.R-project.org/posting-guide.html
 and provide commented, minimal, self-contained, reproducible code.


__
R-help@r-project.org mailing list
https://stat.ethz.ch/mailman/listinfo/r-help
PLEASE do read the posting guide http://www.R-project.org/posting-guide.html
and provide commented, minimal, self-contained, reproducible code.


Re: [R] Non-linear Regression best-fit line

2011-06-18 Thread peter dalgaard

On Jun 17, 2011, at 23:14 , Sean Bignami wrote:

 I am trying to fit a curve to a cumulative mortality curve (logistic) where y 
 is the cumulative proportion of mortalities, and t is the time in hours (see 
 below). Asym. at 0 and 1
 y
 [1] 0. 0.04853859 0.08303777 0.15201970 0.40995074 0.46444992 
 0.62862069 0.95885057 1.
 [10] 1. 1.
 t
 [1]  0 13 20 24 37 42 48 61 72 86 90
 
 I tried to find starting values for a Gompertz non-linear regression by 
 converting the equation (y~1*exp(-beta*exp(-gamma*t)) to a linear form per 
 Dalgaard Introductory Statistics with R pg.279-280. But got this Error 
 message:
 
 lm(log(0-log(y))~t)
 Error in lm.fit(x, y, offset = offset, singular.ok = singular.ok, ...) : 
  NA/NaN/Inf in foreign function call (arg 4)
 
 I tried to change all by 0 and 1 values to non-zero and non-one values (yy 
 and tt below), and was able to get starting estimates.
 
 yy-c(0.1,0.04853859, 0.08303777, 0.15201970, 0.40995074, 
 0.46444992, 0.62862069, 0.95885057, 
 0.99,0.,0.)
 tt-c(0.01,13,20,24,37,42,48,61,72,86,90)
 
 lm(log(0-log(yy))~tt)
 
 Call:
 lm(formula = log(0 - log(yy)) ~ tt)
 
 Coefficients:
 (Intercept)   tt  
 9.5029  -0.3681 
 
 However, when I plug those values into the nls function, I get an error 
 message about the getInitial method
 
 nlsout-nls(y~1*exp(-beta*exp(-gamma*t),start=c(beta=exp(9.5),gamma=.368)));summary(nlsout)
 Error in getInitial.default(func, data, mCall = as.list(match.call(func,  : 
  no 'getInitial' method found for function objects

Not really, but getting your parentheses right on the nls call should at least 
get you closer. There are 3 ( before start but only 1 ), so the start 
bit is part of the formula, etc.

Trying that, I got

 nlsout-nls(yy~1*exp(-beta*exp(-gamma*tt)),start=list(beta=exp(9.5),gamma=.368))
Error in numericDeriv(form[[3L]], names(ind), env) : 
  Missing value or an infinity produced when evaluating the model

so you still have some way to go. (I've been using the yy/tt variables because 
they were easier to paste in from your post. Shouldn't matter much.)

Offhand, I'd say that your procedure for finding starting values is still too 
sensitive to the zeros and ones. If you do 

plot(tt, log(-log(yy)) )

you will see that the last three points (the 1s in your original data) fall way 
off the linear trend. So how about omitting them rather than putting in an 
arbitrary value? And get rid of the zero too.

 lm(log(-log(yy))~tt,subset=2:8)

Call:
lm(formula = log(-log(yy)) ~ tt, subset = 2:8)

Coefficients:
(Intercept)   tt  
 2.5870  -0.0807  


 nlsout-nls(yy~1*exp(-beta*exp(-gamma*tt)),start=list(beta=exp(2.5),gamma=.08))
 summary(nlsout)
Formula: yy ~ 1 * exp(-beta * exp(-gamma * tt))

Parameters:
  Estimate Std. Error t value Pr(|t|)
beta  13.146344.617672.850.019 *  
gamma  0.072840.008698.38  1.5e-05 ***
---
Signif. codes:  0 ‘***’ 0.001 ‘**’ 0.01 ‘*’ 0.05 ‘.’ 0.1 ‘ ’ 1 

Residual standard error: 0.0568 on 9 degrees of freedom

Number of iterations to convergence: 10 
Achieved convergence tolerance: 4.29e-06 



-- 
Peter Dalgaard
Center for Statistics, Copenhagen Business School
Solbjerg Plads 3, 2000 Frederiksberg, Denmark
Phone: (+45)38153501
Email: pd@cbs.dk  Priv: pda...@gmail.com

__
R-help@r-project.org mailing list
https://stat.ethz.ch/mailman/listinfo/r-help
PLEASE do read the posting guide http://www.R-project.org/posting-guide.html
and provide commented, minimal, self-contained, reproducible code.


Re: [R] Non linear Regression: singular gradient matrix at initial parameter estimates

2011-04-12 Thread Peter Ehlers

On 2011-04-11 13:29, Felix Nensa wrote:

Hi,

I am using nls to fit a non linear function to some data but R keeps giving
me singular gradient matrix at initial parameter estimates errors.
For testing purposes I am doing this:

### R code ###

x- 0:140
y- 200 / (1 + exp(17 - x)/2) * exp(-0.02*x) # creating 'perfect' samples
with fitting model
yeps- y + rnorm(length(y), sd = 2) # adding noise

# results in above error
fit = nls(yeps ~ p1 / (1 + exp(p2 - x) / p3) * exp(p4 * x))

###


From what I've found in this list I think that my model is over-parameterized.

How can I work around that?


Take out p3; it's redundant.

Peter Ehlers


Thanks,

Felix

[[alternative HTML version deleted]]

__
R-help@r-project.org mailing list
https://stat.ethz.ch/mailman/listinfo/r-help
PLEASE do read the posting guide http://www.R-project.org/posting-guide.html
and provide commented, minimal, self-contained, reproducible code.


__
R-help@r-project.org mailing list
https://stat.ethz.ch/mailman/listinfo/r-help
PLEASE do read the posting guide http://www.R-project.org/posting-guide.html
and provide commented, minimal, self-contained, reproducible code.


Re: [R] Non linear Regression: singular gradient matrix at initial parameter estimates

2011-04-12 Thread Felix Nensa
Hi Peter,

thank you for your reply. Now I see, that P3 is indeed redundand.
But with the simplified model...

fit = nls(yeps ~ p1 / (1 + exp(p2 - x)) * exp(p4 * x))

...nls still produces the same error.
Any ideas?

Felix

2011/4/12 Peter Ehlers ehl...@ucalgary.ca

 On 2011-04-11 13:29, Felix Nensa wrote:

 Hi,

 I am using nls to fit a non linear function to some data but R keeps
 giving
 me singular gradient matrix at initial parameter estimates errors.
 For testing purposes I am doing this:

 ### R code ###

 x- 0:140
 y- 200 / (1 + exp(17 - x)/2) * exp(-0.02*x) # creating 'perfect' samples
 with fitting model
 yeps- y + rnorm(length(y), sd = 2) # adding noise

 # results in above error
 fit = nls(yeps ~ p1 / (1 + exp(p2 - x) / p3) * exp(p4 * x))

 ###

  From what I've found in this list I think that my model is
 over-parameterized.

 How can I work around that?


 Take out p3; it's redundant.

 Peter Ehlers

  Thanks,

 Felix

[[alternative HTML version deleted]]

 __
 R-help@r-project.org mailing list
 https://stat.ethz.ch/mailman/listinfo/r-help
 PLEASE do read the posting guide
 http://www.R-project.org/posting-guide.html
 and provide commented, minimal, self-contained, reproducible code.




[[alternative HTML version deleted]]

__
R-help@r-project.org mailing list
https://stat.ethz.ch/mailman/listinfo/r-help
PLEASE do read the posting guide http://www.R-project.org/posting-guide.html
and provide commented, minimal, self-contained, reproducible code.


Re: [R] Non linear Regression: singular gradient matrix at initial parameter estimates

2011-04-12 Thread Mario Valle

Use a more realistic starting point instead of the default one:

fit - nls(yeps ~ p1 / (1 + exp(p2 - x)) * exp(p4 * x), 
start=list(p1=410,p2=18,p4=-.03))


This works for me:
 fit
Nonlinear regression model
  model:  yeps ~ p1/(1 + exp(p2 - x)) * exp(p4 * x)
   data:  parent.frame()
   p1p2p4
199.48276  16.28664  -0.01987
 residual sum-of-squares: 560.6

Number of iterations to convergence: 5
Achieved convergence tolerance: 5.637e-07

Ciao!
mario

On 12-Apr-11 18:01, Felix Nensa wrote:

fit = nls(yeps ~ p1 / (1 + exp(p2 - x)) * exp(p4 * x))



--
Ing. Mario Valle
Data Analysis and Visualization Group| http://www.cscs.ch/~mvalle
Swiss National Supercomputing Centre (CSCS)  | Tel:  +41 (91) 610.82.60
v. Cantonale Galleria 2, 6928 Manno, Switzerland | Fax:  +41 (91) 610.82.82

__
R-help@r-project.org mailing list
https://stat.ethz.ch/mailman/listinfo/r-help
PLEASE do read the posting guide http://www.R-project.org/posting-guide.html
and provide commented, minimal, self-contained, reproducible code.


Re: [R] Non linear Regression: singular gradient matrix at initial parameter estimates

2011-04-12 Thread Felix Nensa
Hi Mario,
yes works great. Thanks!

2011/4/12 Mario Valle mva...@cscs.ch

 Use a more realistic starting point instead of the default one:

 fit - nls(yeps ~ p1 / (1 + exp(p2 - x)) * exp(p4 * x),
 start=list(p1=410,p2=18,p4=-.03))

 This works for me:
  fit
 Nonlinear regression model
  model:  yeps ~ p1/(1 + exp(p2 - x)) * exp(p4 * x)
   data:  parent.frame()
   p1p2p4
 199.48276  16.28664  -0.01987
  residual sum-of-squares: 560.6

 Number of iterations to convergence: 5
 Achieved convergence tolerance: 5.637e-07

 Ciao!
mario


 On 12-Apr-11 18:01, Felix Nensa wrote:

 fit = nls(yeps ~ p1 / (1 + exp(p2 - x)) * exp(p4 * x))


 --
 Ing. Mario Valle
 Data Analysis and Visualization Group|
 http://www.cscs.ch/~mvalle
 Swiss National Supercomputing Centre (CSCS)  | Tel:  +41 (91) 610.82.60
 v. Cantonale Galleria 2, 6928 Manno, Switzerland | Fax:  +41 (91) 610.82.82




-- 
Felix Nensa

Luisenstr. 15-17
44787 Bochum
Germany

mail: felix.ne...@googlemail.com
mobile: +49 171 958 51 40

[[alternative HTML version deleted]]

__
R-help@r-project.org mailing list
https://stat.ethz.ch/mailman/listinfo/r-help
PLEASE do read the posting guide http://www.R-project.org/posting-guide.html
and provide commented, minimal, self-contained, reproducible code.


Re: [R] non-linear regression for 3D data

2010-08-15 Thread szisziszilvi

Thanks a lot, this I(xx^2) ... worked.
I guess, I should learn more abot the function poly itself. (so will I... :)
)

Thanks again!
-- 
View this message in context: 
http://r.789695.n4.nabble.com/non-linear-regression-for-3D-data-tp2320982p2325911.html
Sent from the R help mailing list archive at Nabble.com.

__
R-help@r-project.org mailing list
https://stat.ethz.ch/mailman/listinfo/r-help
PLEASE do read the posting guide http://www.R-project.org/posting-guide.html
and provide commented, minimal, self-contained, reproducible code.


Re: [R] non-linear regression for 3D data

2010-08-12 Thread szisziszilvi

I've tried lm, but something is wrong.
I've made a test dataset of 599 data points, my original equation is

zz = 1 +0.5*xx -3.2*xx*xx -1*yy +4.2*yy*yy

but the R gives this result:
---
 mp - read.csv(file=sample.csv,sep=;,header=TRUE)
 lm(zz ~ poly(xx,2) + poly(yy,2), data=mp)

Call:
lm(formula = zz ~ poly(xx, 2) + poly(yy, 2), data = mp)

Coefficients:
 (Intercept)  poly(xx, 2)1  poly(xx, 2)2  poly(yy, 2)1  poly(yy, 2)2  
   25.86  -2239.86   -595.01   2875.54776.84
---
which is definitely not the original. :(

(In case of interest the test dataset is available here:
szisziszilvi.lima-city.de/r)
-- 
View this message in context: 
http://r.789695.n4.nabble.com/non-linear-regression-for-3D-data-tp2320982p2322779.html
Sent from the R help mailing list archive at Nabble.com.

__
R-help@r-project.org mailing list
https://stat.ethz.ch/mailman/listinfo/r-help
PLEASE do read the posting guide http://www.R-project.org/posting-guide.html
and provide commented, minimal, self-contained, reproducible code.


Re: [R] non-linear regression for 3D data

2010-08-12 Thread szisziszilvi

right. How does it come that if I devide the result vector with
10*interception, I get a much better result?

 zz2 - 25.86 -2239.86*mp$xx -595.01*mp$xx*mp$xx + 2875.54*mp$yy +
 776.84*mp$yy*mp$yy
 mp$zz2 - zz2
 library(lattice)
 cloud(zz2/258.6 + zz ~ xx * yy, data=mp)


looks quite pretty.

http://r.789695.n4.nabble.com/file/n2322812/output.jpeg 
-- 
View this message in context: 
http://r.789695.n4.nabble.com/non-linear-regression-for-3D-data-tp2320982p2322812.html
Sent from the R help mailing list archive at Nabble.com.

__
R-help@r-project.org mailing list
https://stat.ethz.ch/mailman/listinfo/r-help
PLEASE do read the posting guide http://www.R-project.org/posting-guide.html
and provide commented, minimal, self-contained, reproducible code.


Re: [R] non-linear regression for 3D data

2010-08-12 Thread Duncan Murdoch

On 12/08/2010 10:35 AM, szisziszilvi wrote:

I've tried lm, but something is wrong.
I've made a test dataset of 599 data points, my original equation is

zz = 1 +0.5*xx -3.2*xx*xx -1*yy +4.2*yy*yy

but the R gives this result:
---
 mp - read.csv(file=sample.csv,sep=;,header=TRUE)
 lm(zz ~ poly(xx,2) + poly(yy,2), data=mp)

Call:
lm(formula = zz ~ poly(xx, 2) + poly(yy, 2), data = mp)

Coefficients:
 (Intercept)  poly(xx, 2)1  poly(xx, 2)2  poly(yy, 2)1  poly(yy, 2)2  
   25.86  -2239.86   -595.01   2875.54776.84

---
which is definitely not the original. :(



I don't think you are interpreting the coefficients properly.  The basis 
functions are orthogonal polynomials, not xx and xx^2, so the 
coefficients won't match the ones you used in your definition.  You 
should compare the predictions of the model, e.g. by looking at


range(predict(lm(zz ~ poly(xx,2) + poly(yy,2), data=mp)) - zz)

If you insist on the power basis, just fit the model as

lm(zz ~ xx + I(xx^2) + yy + I(yy^2), data=mp)

but you might get less accurate predictions due to increased collinearity.

Duncan Murdoch

__
R-help@r-project.org mailing list
https://stat.ethz.ch/mailman/listinfo/r-help
PLEASE do read the posting guide http://www.R-project.org/posting-guide.html
and provide commented, minimal, self-contained, reproducible code.


Re: [R] non-linear regression for 3D data

2010-08-11 Thread Duncan Murdoch

On 11/08/2010 6:15 AM, szisziszilvi wrote:

Hello!

Is there a simplier way in R to get a nonlinear regression (like nls) for a
surface? I have 3D data, and it is definitely not a linear surface with
which it would fit the best. Rather sg like z = a + f(x) + g(y) where
probably both f and g are polinomes (hopefully quadratic).


That's a linear model if f and g are polynomials.  You can fit it with

lm(z ~ poly(x, 2) + poly(y, 2))

The 2's are the degrees of each polynomial; the intercept a is implied.

Duncan Murdoch

__
R-help@r-project.org mailing list
https://stat.ethz.ch/mailman/listinfo/r-help
PLEASE do read the posting guide http://www.R-project.org/posting-guide.html
and provide commented, minimal, self-contained, reproducible code.


Re: [R] non-linear regression for 3D data

2010-08-11 Thread szisziszilvi

oh, god, please don't tell anybody...
-- 
View this message in context: 
http://r.789695.n4.nabble.com/non-linear-regression-for-3D-data-tp2320982p2321082.html
Sent from the R help mailing list archive at Nabble.com.

__
R-help@r-project.org mailing list
https://stat.ethz.ch/mailman/listinfo/r-help
PLEASE do read the posting guide http://www.R-project.org/posting-guide.html
and provide commented, minimal, self-contained, reproducible code.


Re: [R] Non-linear regression

2010-02-07 Thread David Winsemius
It appears my suspicions about this being homework were unfounded.  
Given the additional problems with excess zeroes, you may want to  
examine the extremely informative material on analysis of such  
problems written by Zeileis, Kleiber and Jackman:
(easily found in case you have misplaced it, as I had, with a Google  
search for:

r-project zero-inflated hurdle models

Regression Models for Count Data in R
http://cran.cnr.berkeley.edu/web/packages/pscl/vignettes/countreg.pdf

--
David.

On Feb 6, 2010, at 10:56 PM, kupz wrote:



Agreed, it would be simple to propose the relationship, however the
regression is necessary to model the data properly. Unfortunately a  
simple
decay based on those two points does not have the proper shape  
necessary.
This is due to an extreme amount of zero inflation with this  
fisheries data.


On another note, I have a working solution for the problem, I am  
excluding a
portion of the zero data based on some other apriori assumptions..  
Thanks

for your help though.
--
View this message in context: 
http://n4.nabble.com/Non-linear-regression-tp1471736p1471749.html
Sent from the R help mailing list archive at Nabble.com.

__
R-help@r-project.org mailing list
https://stat.ethz.ch/mailman/listinfo/r-help
PLEASE do read the posting guide http://www.R-project.org/posting-guide.html
and provide commented, minimal, self-contained, reproducible code.


David Winsemius, MD
Heritage Laboratories
West Hartford, CT

__
R-help@r-project.org mailing list
https://stat.ethz.ch/mailman/listinfo/r-help
PLEASE do read the posting guide http://www.R-project.org/posting-guide.html
and provide commented, minimal, self-contained, reproducible code.


Re: [R] Non-linear regression

2010-02-06 Thread David Winsemius


On Feb 6, 2010, at 10:33 PM, kupz wrote:



So I have a data set I would like to model using a non-linear  
method. I know
it should be an exponential decay. However I know what the first  
derivative
of the equation should be at two points, x=0 and x=100. Is there  
anyway to
establish this when inputing the model or how would one go about  
this before

the nls statement


Given the rather simple relationship between the exponential function  
and its derivative, why would you need the regression if you already  
have those two points for dy/dx ( as well as the value of the function  
at x=0)? Is this homework?


--

David Winsemius, MD
Heritage Laboratories
West Hartford, CT

__
R-help@r-project.org mailing list
https://stat.ethz.ch/mailman/listinfo/r-help
PLEASE do read the posting guide http://www.R-project.org/posting-guide.html
and provide commented, minimal, self-contained, reproducible code.


Re: [R] Non-linear regression

2010-02-06 Thread kupz

Agreed, it would be simple to propose the relationship, however the
regression is necessary to model the data properly. Unfortunately a simple
decay based on those two points does not have the proper shape necessary.
This is due to an extreme amount of zero inflation with this fisheries data. 

On another note, I have a working solution for the problem, I am excluding a
portion of the zero data based on some other apriori assumptions.. Thanks
for your help though. 
-- 
View this message in context: 
http://n4.nabble.com/Non-linear-regression-tp1471736p1471749.html
Sent from the R help mailing list archive at Nabble.com.

__
R-help@r-project.org mailing list
https://stat.ethz.ch/mailman/listinfo/r-help
PLEASE do read the posting guide http://www.R-project.org/posting-guide.html
and provide commented, minimal, self-contained, reproducible code.


Re: [R] non-linear regression

2009-12-11 Thread Alane

Katharine,

The problem of estimation of parameters in R is that you have to know the 
value of the initial estimates very accurately, otherwise it does not 
converge.


The example below could be resolved in Excel, however in  does not converge. 
How to solve the problem?


I made the chart on a logarithmic scale to better visualize the differences.

Send the data file attached.

The commands are below:

tx.br - read.table('c:/tx.br.H.txt',header=F,dec=',')
tx.br -tx.br[,1]
id-1:100

qx.suav - function(id,A,B,C,D,E,F,G,H,K)
 (A^((id+B)^C)+(D*exp(-E*(log(id)-log(F))^2))+(G*H^id)/(1+(K*G*H^id)))

HP - nls(tx.br~qx.suav(id,A,B,C,D,E,F,G,H,K),
 data=data.frame(id=id,tx.br=tx.br),
 trace=TRUE,nls.control(maxiter=5,warnOnly=TRUE,minFactor = 
0.1),

algorithm='port',
start=list(A=0.000644,B=0.016761290,C=0.10927095582,D=0.00094877,
  
E=5.949082737,F=24.526811,G=0.46733960,H=1.0970550987,K=0.771722501657),
 lower=list(A=0,B=0,C=0,D=0,E=0,F=0,G=0,H=0,K=0))

HP

matplot(cbind(log(fitted(HP)), log(tx.br)),type=l)



- Original Message - 
From: Katharine Mullen k...@few.vu.nl

To: AneSR citb...@terra.com.br
Cc: r-help@r-project.org
Sent: Thursday, December 10, 2009 9:55 PM
Subject: Re: [R] non-linear regression



You did not provide the data or a way of generating it.

I would guess that Excel finds the same solution (the same residual sum-of
squares) as nls but that it uses a weak convergence criterion and/or does
not give you information regarding why it terminates.

Regarding the step size:  you can set the minimum step size factor via the
minFactor argument of control.

qx.suav - function(id,A,B,C,D,E,F,G,H,K)
 (A^((id+B)^C)+(D*exp(-E*(log(id)-log(F))^2))+(G*H^id)/(1+(K*G*H^id)))

## make noisy data from model
id - 1:1000
tx.br - qx.suav(id,A=0.0006347,B=0.0453814,C=0.1353538,D=0.1353538,
E=0.0002127,F=38.5448420,G=0.115,H=1.1109286,
K=0.382070638)
set.seed(1)
tx.br - tx.br + rnorm(length(tx.br),0,.0001)

HP - nls(tx.br~qx.suav(id,A,B,C,D,E,F,G,H,K),
 data=data.frame(id=id,tx.br=tx.br),
 trace=TRUE,nls.control(maxiter=5000,warnOnly=TRUE),
 algorithm='port',
 start=list(A=0.0006347,B=0.0453814,C=0.1353538,D=0.1353538,
   E=0.0002127,F=38.5448420,G=0.115,H=1.1109286,
   K=0.382070638),
 lower=list(A=0,B=0,C=0,D=0,E=0,F=0,G=0,H=0,K=0))
matplot(cbind(fitted(HP), tx.br),type=l)

On Thu, 10 Dec 2009, AneSR wrote:



I have a non-linear regression with 8 parameters to solve  however it
does not converge ... easily solves the excel ... including the initial
estimates used in the R were found in the excel ... Another question is 
how

to establish the increments of R by the parameters in the search ..


qx.suav-function(id,A,B,C,D,E,F,G,H,K){(A^((id+B)^C)+(D*exp(-E*(log(id)-log(F))^2))+(G*H^id)/(1+(K*G*H^id)))}
HP-nls(tx.br~qx.suav(id,A,B,C,D,E,F,G,H,K),data=data.frame(id=id,tx.br=tx.br),
trace=TRUE,nls.control(maxiter=5000),algorithm='port',start=list(A=0.0006347,B=0.0453814,C=0.1353538,D=0.1353538,E=0.0002127,F=38.5448420,G=0.115,H=1.1109286,K=0.382070638),lower=list(A=0,B=0,C=0,D=0,E=0,F=0,G=0,H=0,K=0))
summary(HP)

How to solve this problem in R?

Ane
--
View this message in context: 
http://n4.nabble.com/non-linear-regression-tp959977p959977.html

Sent from the R help mailing list archive at Nabble.com.

[[alternative HTML version deleted]]

__
R-help@r-project.org mailing list
https://stat.ethz.ch/mailman/listinfo/r-help
PLEASE do read the posting guide 
http://www.R-project.org/posting-guide.html

and provide commented, minimal, self-contained, reproducible code.










No virus found in this incoming message.
Checked by AVG - www.avg.com

07:36:00

0,000757
0,000356
0,000290
0,000232
0,000199
0,000283
0,000128
0,000235
0,000230
0,000179
0,000197
0,000172
0,000294
0,000307
0,000411
0,000619
0,000812
0,000949
0,001116
0,001134
0,001159
0,001151
0,001321
0,001235
0,001296
0,001287
0,001241
0,001469
0,001278
0,001466
0,001440
0,001881
0,001457
0,001843
0,001754
0,001796
0,001915
0,002199
0,002120
0,002471
0,002213
0,002478
0,002975
0,002583
0,002751
0,003388
0,003956
0,004369
0,004062
0,004410
0,004939
0,005492
0,005961
0,006311
0,006846
0,007637
0,008968
0,010612
0,010907
0,011986
0,013174
0,014316
0,015557
0,017824
0,018740
0,020338
0,022034
0,022339
0,025929
0,030967
0,033520
0,039556
0,041168
0,044811
0,049932
0,047401
0,060318
0,063559
0,064084
0,077614
0,080425
0,094073
0,102383
0,116288
0,112727
0,114192
0,137330
0,159065
0,138378
0,135844
0,148244
0,165927
0,180803
0,217185
0,241570
0,249358
0,309202
0,269479
0,325142
0,461620
__
R-help@r-project.org mailing list
https://stat.ethz.ch/mailman/listinfo/r-help
PLEASE do read the posting guide http://www.R-project.org/posting

Re: [R] non-linear regression

2009-12-11 Thread Katharine Mullen

 The problem of estimation of parameters in R is that you have to know the
 value of the initial estimates very accurately, otherwise it does not
 converge.

This was discussed on R-help in the last 2 weeks; see the thread on
'Starting estimates for nls Exponential Fit'.


 The example below could be resolved in Excel, however in  does not converge.
 How to solve the problem?

Can you consider a model with a different functional form or do you need
to fit this exact function?

If you want to minimize log(data) log(model) RSS, then:

tx.br - read.table('tx.br.H.txt',header=F,dec=',')
tx.br - tx.br[,1]
id - 1:100

qx.suav - function(id,A,B,C,D,E,F,G,H,K)
  (A^((id+B)^C)+(D*exp(-E*(log(id)-log(F))^2))+(G*H^id)/(1+(K*G*H^id)))

newD - log(tx.br)

HP - nls(newD~log(qx.suav(id,A,B,C,D,E,F,G,H,K)),
  data=data.frame(id=id,newD=newD),
  trace=TRUE,
  nls.control(maxiter=5,warnOnly=TRUE),
  algorithm='port',
  start=list(A=0.000644,B=0.016761290,C=0.10927095582,D=0.00094877,
E=5.949082737,F=24.526811,G=0.46733960,H=1.0970550987,
K=0.771722501657),
  lower=list(A=0,B=0,C=0,D=0,E=0,F=0,G=0,H=0,K=0))

HP

par(mfrow=c(2,1))
matplot(cbind(fitted(HP), newD),type=l, main=model and fit)

matplot(cbind(exp(fitted(HP)), exp(newD)), type=l,
main=transformed back to original space)

 I made the chart on a logarithmic scale to better visualize the differences.

 Send the data file attached.

 The commands are below:

 tx.br - read.table('c:/tx.br.H.txt',header=F,dec=',')
 tx.br -tx.br[,1]
 id-1:100

 qx.suav - function(id,A,B,C,D,E,F,G,H,K)
   (A^((id+B)^C)+(D*exp(-E*(log(id)-log(F))^2))+(G*H^id)/(1+(K*G*H^id)))

 HP - nls(tx.br~qx.suav(id,A,B,C,D,E,F,G,H,K),
   data=data.frame(id=id,tx.br=tx.br),
   trace=TRUE,nls.control(maxiter=5,warnOnly=TRUE,minFactor =
 0.1),
  algorithm='port',
  start=list(A=0.000644,B=0.016761290,C=0.10927095582,D=0.00094877,

 E=5.949082737,F=24.526811,G=0.46733960,H=1.0970550987,K=0.771722501657),
   lower=list(A=0,B=0,C=0,D=0,E=0,F=0,G=0,H=0,K=0))

 HP

 matplot(cbind(log(fitted(HP)), log(tx.br)),type=l)



 - Original Message -
 From: Katharine Mullen k...@few.vu.nl
 To: AneSR citb...@terra.com.br
 Cc: r-help@r-project.org
 Sent: Thursday, December 10, 2009 9:55 PM
 Subject: Re: [R] non-linear regression


  You did not provide the data or a way of generating it.
 
  I would guess that Excel finds the same solution (the same residual sum-of
  squares) as nls but that it uses a weak convergence criterion and/or does
  not give you information regarding why it terminates.
 
  Regarding the step size:  you can set the minimum step size factor via the
  minFactor argument of control.
 
  qx.suav - function(id,A,B,C,D,E,F,G,H,K)
   (A^((id+B)^C)+(D*exp(-E*(log(id)-log(F))^2))+(G*H^id)/(1+(K*G*H^id)))
 
  ## make noisy data from model
  id - 1:1000
  tx.br - qx.suav(id,A=0.0006347,B=0.0453814,C=0.1353538,D=0.1353538,
  E=0.0002127,F=38.5448420,G=0.115,H=1.1109286,
  K=0.382070638)
  set.seed(1)
  tx.br - tx.br + rnorm(length(tx.br),0,.0001)
 
  HP - nls(tx.br~qx.suav(id,A,B,C,D,E,F,G,H,K),
   data=data.frame(id=id,tx.br=tx.br),
   trace=TRUE,nls.control(maxiter=5000,warnOnly=TRUE),
   algorithm='port',
   start=list(A=0.0006347,B=0.0453814,C=0.1353538,D=0.1353538,
 E=0.0002127,F=38.5448420,G=0.115,H=1.1109286,
 K=0.382070638),
   lower=list(A=0,B=0,C=0,D=0,E=0,F=0,G=0,H=0,K=0))
  matplot(cbind(fitted(HP), tx.br),type=l)
 
  On Thu, 10 Dec 2009, AneSR wrote:
 
 
  I have a non-linear regression with 8 parameters to solve  however it
  does not converge ... easily solves the excel ... including the initial
  estimates used in the R were found in the excel ... Another question is
  how
  to establish the increments of R by the parameters in the search ..
 
 
  qx.suav-function(id,A,B,C,D,E,F,G,H,K){(A^((id+B)^C)+(D*exp(-E*(log(id)-log(F))^2))+(G*H^id)/(1+(K*G*H^id)))}
  HP-nls(tx.br~qx.suav(id,A,B,C,D,E,F,G,H,K),data=data.frame(id=id,tx.br=tx.br),
  trace=TRUE,nls.control(maxiter=5000),algorithm='port',start=list(A=0.0006347,B=0.0453814,C=0.1353538,D=0.1353538,E=0.0002127,F=38.5448420,G=0.115,H=1.1109286,K=0.382070638),lower=list(A=0,B=0,C=0,D=0,E=0,F=0,G=0,H=0,K=0))
  summary(HP)
 
  How to solve this problem in R?
 
  Ane
  --
  View this message in context:
  http://n4.nabble.com/non-linear-regression-tp959977p959977.html
  Sent from the R help mailing list archive at Nabble.com.
 
  [[alternative HTML version deleted]]
 
  __
  R-help@r-project.org mailing list
  https://stat.ethz.ch/mailman/listinfo/r-help
  PLEASE do read the posting guide
  http://www.R-project.org/posting-guide.html
  and provide commented, minimal, self-contained, reproducible code

Re: [R] non-linear regression

2009-12-10 Thread Katharine Mullen
You did not provide the data or a way of generating it.

I would guess that Excel finds the same solution (the same residual sum-of
squares) as nls but that it uses a weak convergence criterion and/or does
not give you information regarding why it terminates.

Regarding the step size:  you can set the minimum step size factor via the
minFactor argument of control.

qx.suav - function(id,A,B,C,D,E,F,G,H,K)
  (A^((id+B)^C)+(D*exp(-E*(log(id)-log(F))^2))+(G*H^id)/(1+(K*G*H^id)))

## make noisy data from model
id - 1:1000
tx.br - qx.suav(id,A=0.0006347,B=0.0453814,C=0.1353538,D=0.1353538,
 E=0.0002127,F=38.5448420,G=0.115,H=1.1109286,
 K=0.382070638)
set.seed(1)
tx.br - tx.br + rnorm(length(tx.br),0,.0001)

HP - nls(tx.br~qx.suav(id,A,B,C,D,E,F,G,H,K),
  data=data.frame(id=id,tx.br=tx.br),
  trace=TRUE,nls.control(maxiter=5000,warnOnly=TRUE),
  algorithm='port',
  start=list(A=0.0006347,B=0.0453814,C=0.1353538,D=0.1353538,
E=0.0002127,F=38.5448420,G=0.115,H=1.1109286,
K=0.382070638),
  lower=list(A=0,B=0,C=0,D=0,E=0,F=0,G=0,H=0,K=0))
matplot(cbind(fitted(HP), tx.br),type=l)

On Thu, 10 Dec 2009, AneSR wrote:


 I have a non-linear regression with 8 parameters to solve  however it
 does not converge ... easily solves the excel ... including the initial
 estimates used in the R were found in the excel ... Another question is how
 to establish the increments of R by the parameters in the search ..


 qx.suav-function(id,A,B,C,D,E,F,G,H,K){(A^((id+B)^C)+(D*exp(-E*(log(id)-log(F))^2))+(G*H^id)/(1+(K*G*H^id)))}
 HP-nls(tx.br~qx.suav(id,A,B,C,D,E,F,G,H,K),data=data.frame(id=id,tx.br=tx.br),
 trace=TRUE,nls.control(maxiter=5000),algorithm='port',start=list(A=0.0006347,B=0.0453814,C=0.1353538,D=0.1353538,E=0.0002127,F=38.5448420,G=0.115,H=1.1109286,K=0.382070638),lower=list(A=0,B=0,C=0,D=0,E=0,F=0,G=0,H=0,K=0))
 summary(HP)

 How to solve this problem in R?

 Ane
 --
 View this message in context: 
 http://n4.nabble.com/non-linear-regression-tp959977p959977.html
 Sent from the R help mailing list archive at Nabble.com.

   [[alternative HTML version deleted]]

 __
 R-help@r-project.org mailing list
 https://stat.ethz.ch/mailman/listinfo/r-help
 PLEASE do read the posting guide http://www.R-project.org/posting-guide.html
 and provide commented, minimal, self-contained, reproducible code.


__
R-help@r-project.org mailing list
https://stat.ethz.ch/mailman/listinfo/r-help
PLEASE do read the posting guide http://www.R-project.org/posting-guide.html
and provide commented, minimal, self-contained, reproducible code.


Re: [R] Non-Linear Regression with two Predictors

2009-07-27 Thread Berlinerfee
Hi and thank you for your reply,

in my new regression formular the parameter delta is inserted:
fit - nls(dataset$V2~(( alpha / ( 1 + exp( beta - gamma* dataset$V1 ) ) 
) + (dataset$V6*delta)),data=dataset,start=startparam)

The sense is, that dataset$V6 is a dummy variable that represents the 
german reunion. I expect, that the delta in the regression is about 
1.000.000 because of the reunion. The logistic function thus has a jump 
at this point. But I would like to get the exact paramter value for 
delta (the jump) as the parameters for the logistic function of growth 
(alpha to gamma). The partial derivative to delta would be like a 
stair-function. It is 0 until 1990 and 1 there after.

Any idea? Thank you!

Regards

Moshe Olshansky schrieb:
 Hi,

 I believe that since delta does not appear in the function you are 
 optimizing, it's partial derivative with respect to delta is always 0 and so 
 the gradient is singular.
 Why do you need delta at all?

 --- On Mon, 27/7/09, Berlinerfee berliner...@yahoo.de wrote:

   
 From: Berlinerfee berliner...@yahoo.de
 Subject: [R] Non-Linear Regression with two Predictors
 To: r-h...@stat.math.ethz.ch
 Received: Monday, 27 July, 2009, 2:52 AM
 Hello there,

 I am using nls the first time for a non-linear regression
 with a logistic growth function:
 startparam - c(alpha=3e+07,beta=4000,gamma=2)
 fit - nls(dataset$V2~(( alpha / ( 1 + exp( beta - gamma
 * dataset$V1 ) ) ) ),data=dataset,start=startparam)

 Everything works fine and i get good results. Now I would
 like to improve the results using my DUMMY Variable
 (dataset$V6) the runs half of the time 0 and then 1. This is
 my new nls:
 startparam -
 c(alpha=3e+07,beta=4000,gamma=2,delta=100)
 fit - nls(dataset$V2~(( alpha / ( 1 + exp( beta - gamma
 * dataset$V1 ) ) ) + (dataset$V6*dataset$V1*delta)
 ),data=dataset,start=startparam)

 I get Singular Gradient Matrice. May anyone give me the
 right nls function for this problem??

 Regards

 __
 R-help@r-project.org
 mailing list
 https://stat.ethz.ch/mailman/listinfo/r-help
 PLEASE do read the posting guide http://www.R-project.org/posting-guide.html
 and provide commented, minimal, self-contained,
 reproducible code.

 

   


[[alternative HTML version deleted]]

__
R-help@r-project.org mailing list
https://stat.ethz.ch/mailman/listinfo/r-help
PLEASE do read the posting guide http://www.R-project.org/posting-guide.html
and provide commented, minimal, self-contained, reproducible code.


Re: [R] Non-linear regression/Quantile regression

2009-06-09 Thread Gabor Grothendieck
Those are linear in the coefficients so try these:

library(quantreg)

rq1 - rq(demand ~ Time + I(Time^2), data = BOD, tau= 1:3/4); rq1

# or
rq2 - rq(demand ~ poly(Time, 2), data = BOD, tau = 1:3/4); rq2


On Tue, Jun 9, 2009 at 10:55 AM, despairedmeyfa...@uni-potsdam.de wrote:

 Hi,

 I'm relatively new to R and need to do a quantile regression. Linear
 quantile regression works, but for my data I need some quadratic function.
 So I guess, I have to use a nonlinear quantile regression. I tried the
 example on the help page for nlrq with my data and it worked. But the
 example there was with a SSlogis model. Trying to write

 dat.nlrq - nlrq(BM ~ I(Regen100^2), data=dat, tau=0.25, trace=TRUE)

 or

 dat.nlrq - nlrq(BM ~ poly(Regen100^2), data=dat, tau=0.25, trace=TRUE)

 (I don't know the difference) both gave me the following error message:

 error in getInitial.default(func, data, mCall = as.list(match.call(func,  :
  no 'getInitial' method found for function objects

 Looking in getInitial, it must have to do something with the starting
 parameters or selfStart model. But I have no idea, what this is and how I
 handle this problem. Can anyone please help?

 Thanks a lot in advance!
 --
 View this message in context: 
 http://www.nabble.com/Non-linear-regression-Quantile-regression-tp23944530p23944530.html
 Sent from the R help mailing list archive at Nabble.com.

 __
 R-help@r-project.org mailing list
 https://stat.ethz.ch/mailman/listinfo/r-help
 PLEASE do read the posting guide http://www.R-project.org/posting-guide.html
 and provide commented, minimal, self-contained, reproducible code.


__
R-help@r-project.org mailing list
https://stat.ethz.ch/mailman/listinfo/r-help
PLEASE do read the posting guide http://www.R-project.org/posting-guide.html
and provide commented, minimal, self-contained, reproducible code.


Re: [R] Non-linear regression/Quantile regression

2009-06-09 Thread despaired

Hi,

thanks, it works :-)
But where is the difference between demand ~ Time + I(Time^2) and demand ~
poly(Time, 2) ?
Or: How do I have to interpret the results? (I get different results for the
two methods)

Thank you again!


Gabor Grothendieck wrote:
 
 Those are linear in the coefficients so try these:
 
 library(quantreg)
 
 rq1 - rq(demand ~ Time + I(Time^2), data = BOD, tau= 1:3/4); rq1
 
 # or
 rq2 - rq(demand ~ poly(Time, 2), data = BOD, tau = 1:3/4); rq2
 
 
 On Tue, Jun 9, 2009 at 10:55 AM, despairedmeyfa...@uni-potsdam.de wrote:

 Hi,

 I'm relatively new to R and need to do a quantile regression. Linear
 quantile regression works, but for my data I need some quadratic
 function.
 So I guess, I have to use a nonlinear quantile regression. I tried the
 example on the help page for nlrq with my data and it worked. But the
 example there was with a SSlogis model. Trying to write

 dat.nlrq - nlrq(BM ~ I(Regen100^2), data=dat, tau=0.25, trace=TRUE)

 or

 dat.nlrq - nlrq(BM ~ poly(Regen100^2), data=dat, tau=0.25, trace=TRUE)

 (I don't know the difference) both gave me the following error message:

 error in getInitial.default(func, data, mCall = as.list(match.call(func,
  :
  no 'getInitial' method found for function objects

 Looking in getInitial, it must have to do something with the starting
 parameters or selfStart model. But I have no idea, what this is and how I
 handle this problem. Can anyone please help?

 Thanks a lot in advance!
 --
 View this message in context:
 http://www.nabble.com/Non-linear-regression-Quantile-regression-tp23944530p23944530.html
 Sent from the R help mailing list archive at Nabble.com.

 __
 R-help@r-project.org mailing list
 https://stat.ethz.ch/mailman/listinfo/r-help
 PLEASE do read the posting guide
 http://www.R-project.org/posting-guide.html
 and provide commented, minimal, self-contained, reproducible code.

 
 __
 R-help@r-project.org mailing list
 https://stat.ethz.ch/mailman/listinfo/r-help
 PLEASE do read the posting guide
 http://www.R-project.org/posting-guide.html
 and provide commented, minimal, self-contained, reproducible code.
 
 

-- 
View this message in context: 
http://www.nabble.com/Non-linear-regression-Quantile-regression-tp23944530p23945900.html
Sent from the R help mailing list archive at Nabble.com.

__
R-help@r-project.org mailing list
https://stat.ethz.ch/mailman/listinfo/r-help
PLEASE do read the posting guide http://www.R-project.org/posting-guide.html
and provide commented, minimal, self-contained, reproducible code.


Re: [R] Non-linear regression/Quantile regression

2009-06-09 Thread Ravi Varadhan

Try  `poly(Time, 2, raw=TRUE)'

Here is an example:

Time - runif(100)

demand - 1 + 0.5 * Time - 1.2 * Time^2 + rt(100, df=4)

rq1 - rq(demand ~ Time + I(Time^2), tau = 1:3/4) 

rq2 - rq(demand ~ poly(Time, 2, raw=TRUE), tau = 1:3/4) 

all.equal(c(rq1$coef), c(rq2$coef))


Ravi.


---

Ravi Varadhan, Ph.D.

Assistant Professor, The Center on Aging and Health

Division of Geriatric Medicine and Gerontology 

Johns Hopkins University

Ph: (410) 502-2619

Fax: (410) 614-9625

Email: rvarad...@jhmi.edu

Webpage:
http://www.jhsph.edu/agingandhealth/People/Faculty_personal_pages/Varadhan.h
tml







-Original Message-
From: r-help-boun...@r-project.org [mailto:r-help-boun...@r-project.org] On
Behalf Of despaired
Sent: Tuesday, June 09, 2009 11:59 AM
To: r-help@r-project.org
Subject: Re: [R] Non-linear regression/Quantile regression


Hi,

thanks, it works :-)
But where is the difference between demand ~ Time + I(Time^2) and demand ~
poly(Time, 2) ?
Or: How do I have to interpret the results? (I get different results for the
two methods)

Thank you again!


Gabor Grothendieck wrote:
 
 Those are linear in the coefficients so try these:
 
 library(quantreg)
 
 rq1 - rq(demand ~ Time + I(Time^2), data = BOD, tau= 1:3/4); rq1
 
 # or
 rq2 - rq(demand ~ poly(Time, 2), data = BOD, tau = 1:3/4); rq2
 
 
 On Tue, Jun 9, 2009 at 10:55 AM, despairedmeyfa...@uni-potsdam.de wrote:

 Hi,

 I'm relatively new to R and need to do a quantile regression. Linear 
 quantile regression works, but for my data I need some quadratic 
 function.
 So I guess, I have to use a nonlinear quantile regression. I tried 
 the example on the help page for nlrq with my data and it worked. But 
 the example there was with a SSlogis model. Trying to write

 dat.nlrq - nlrq(BM ~ I(Regen100^2), data=dat, tau=0.25, trace=TRUE)

 or

 dat.nlrq - nlrq(BM ~ poly(Regen100^2), data=dat, tau=0.25, 
 trace=TRUE)

 (I don't know the difference) both gave me the following error message:

 error in getInitial.default(func, data, mCall = 
 as.list(match.call(func,
  :
  no 'getInitial' method found for function objects

 Looking in getInitial, it must have to do something with the starting 
 parameters or selfStart model. But I have no idea, what this is and 
 how I handle this problem. Can anyone please help?

 Thanks a lot in advance!
 --
 View this message in context:
 http://www.nabble.com/Non-linear-regression-Quantile-regression-tp239
 44530p23944530.html Sent from the R help mailing list archive at 
 Nabble.com.

 __
 R-help@r-project.org mailing list
 https://stat.ethz.ch/mailman/listinfo/r-help
 PLEASE do read the posting guide
 http://www.R-project.org/posting-guide.html
 and provide commented, minimal, self-contained, reproducible code.

 
 __
 R-help@r-project.org mailing list
 https://stat.ethz.ch/mailman/listinfo/r-help
 PLEASE do read the posting guide
 http://www.R-project.org/posting-guide.html
 and provide commented, minimal, self-contained, reproducible code.
 
 

--
View this message in context:
http://www.nabble.com/Non-linear-regression-Quantile-regression-tp23944530p2
3945900.html
Sent from the R help mailing list archive at Nabble.com.

__
R-help@r-project.org mailing list
https://stat.ethz.ch/mailman/listinfo/r-help
PLEASE do read the posting guide http://www.R-project.org/posting-guide.html
and provide commented, minimal, self-contained, reproducible code.

__
R-help@r-project.org mailing list
https://stat.ethz.ch/mailman/listinfo/r-help
PLEASE do read the posting guide http://www.R-project.org/posting-guide.html
and provide commented, minimal, self-contained, reproducible code.


Re: [R] Non-linear regression/Quantile regression

2009-06-09 Thread Greg Snow
poly by default uses orthogonal polynomials which work better mathematically 
but are harder to interpret.  See ?poly
 
-- 
Gregory (Greg) L. Snow Ph.D.
Statistical Data Center
Intermountain Healthcare
greg.s...@imail.org
801.408.8111


 -Original Message-
 From: r-help-boun...@r-project.org [mailto:r-help-boun...@r-
 project.org] On Behalf Of despaired
 Sent: Tuesday, June 09, 2009 9:59 AM
 To: r-help@r-project.org
 Subject: Re: [R] Non-linear regression/Quantile regression
 
 
 Hi,
 
 thanks, it works :-)
 But where is the difference between demand ~ Time + I(Time^2) and
 demand ~
 poly(Time, 2) ?
 Or: How do I have to interpret the results? (I get different results
 for the
 two methods)
 
 Thank you again!
 
 
 Gabor Grothendieck wrote:
 
  Those are linear in the coefficients so try these:
 
  library(quantreg)
 
  rq1 - rq(demand ~ Time + I(Time^2), data = BOD, tau= 1:3/4); rq1
 
  # or
  rq2 - rq(demand ~ poly(Time, 2), data = BOD, tau = 1:3/4); rq2
 
 
  On Tue, Jun 9, 2009 at 10:55 AM, despairedmeyfa...@uni-potsdam.de
 wrote:
 
  Hi,
 
  I'm relatively new to R and need to do a quantile regression. Linear
  quantile regression works, but for my data I need some quadratic
  function.
  So I guess, I have to use a nonlinear quantile regression. I tried
 the
  example on the help page for nlrq with my data and it worked. But
 the
  example there was with a SSlogis model. Trying to write
 
  dat.nlrq - nlrq(BM ~ I(Regen100^2), data=dat, tau=0.25, trace=TRUE)
 
  or
 
  dat.nlrq - nlrq(BM ~ poly(Regen100^2), data=dat, tau=0.25,
 trace=TRUE)
 
  (I don't know the difference) both gave me the following error
 message:
 
  error in getInitial.default(func, data, mCall =
 as.list(match.call(func,
   :
   no 'getInitial' method found for function objects
 
  Looking in getInitial, it must have to do something with the
 starting
  parameters or selfStart model. But I have no idea, what this is and
 how I
  handle this problem. Can anyone please help?
 
  Thanks a lot in advance!
  --
  View this message in context:
  http://www.nabble.com/Non-linear-regression-Quantile-regression-
 tp23944530p23944530.html
  Sent from the R help mailing list archive at Nabble.com.
 
  __
  R-help@r-project.org mailing list
  https://stat.ethz.ch/mailman/listinfo/r-help
  PLEASE do read the posting guide
  http://www.R-project.org/posting-guide.html
  and provide commented, minimal, self-contained, reproducible code.
 
 
  __
  R-help@r-project.org mailing list
  https://stat.ethz.ch/mailman/listinfo/r-help
  PLEASE do read the posting guide
  http://www.R-project.org/posting-guide.html
  and provide commented, minimal, self-contained, reproducible code.
 
 
 
 --
 View this message in context: http://www.nabble.com/Non-linear-
 regression-Quantile-regression-tp23944530p23945900.html
 Sent from the R help mailing list archive at Nabble.com.
 
 __
 R-help@r-project.org mailing list
 https://stat.ethz.ch/mailman/listinfo/r-help
 PLEASE do read the posting guide http://www.R-project.org/posting-
 guide.html
 and provide commented, minimal, self-contained, reproducible code.
__
R-help@r-project.org mailing list
https://stat.ethz.ch/mailman/listinfo/r-help
PLEASE do read the posting guide http://www.R-project.org/posting-guide.html
and provide commented, minimal, self-contained, reproducible code.


Re: [R] Non-linear regression/Quantile regression

2009-06-09 Thread Gabor Grothendieck
The coefficients are different but the predictions are the same.

On Tue, Jun 9, 2009 at 12:41 PM, Greg Snowgreg.s...@imail.org wrote:
 poly by default uses orthogonal polynomials which work better mathematically 
 but are harder to interpret.  See ?poly

 --
 Gregory (Greg) L. Snow Ph.D.
 Statistical Data Center
 Intermountain Healthcare
 greg.s...@imail.org
 801.408.8111


 -Original Message-
 From: r-help-boun...@r-project.org [mailto:r-help-boun...@r-
 project.org] On Behalf Of despaired
 Sent: Tuesday, June 09, 2009 9:59 AM
 To: r-help@r-project.org
 Subject: Re: [R] Non-linear regression/Quantile regression


 Hi,

 thanks, it works :-)
 But where is the difference between demand ~ Time + I(Time^2) and
 demand ~
 poly(Time, 2) ?
 Or: How do I have to interpret the results? (I get different results
 for the
 two methods)

 Thank you again!


 Gabor Grothendieck wrote:
 
  Those are linear in the coefficients so try these:
 
  library(quantreg)
 
  rq1 - rq(demand ~ Time + I(Time^2), data = BOD, tau= 1:3/4); rq1
 
  # or
  rq2 - rq(demand ~ poly(Time, 2), data = BOD, tau = 1:3/4); rq2
 
 
  On Tue, Jun 9, 2009 at 10:55 AM, despairedmeyfa...@uni-potsdam.de
 wrote:
 
  Hi,
 
  I'm relatively new to R and need to do a quantile regression. Linear
  quantile regression works, but for my data I need some quadratic
  function.
  So I guess, I have to use a nonlinear quantile regression. I tried
 the
  example on the help page for nlrq with my data and it worked. But
 the
  example there was with a SSlogis model. Trying to write
 
  dat.nlrq - nlrq(BM ~ I(Regen100^2), data=dat, tau=0.25, trace=TRUE)
 
  or
 
  dat.nlrq - nlrq(BM ~ poly(Regen100^2), data=dat, tau=0.25,
 trace=TRUE)
 
  (I don't know the difference) both gave me the following error
 message:
 
  error in getInitial.default(func, data, mCall =
 as.list(match.call(func,
   :
   no 'getInitial' method found for function objects
 
  Looking in getInitial, it must have to do something with the
 starting
  parameters or selfStart model. But I have no idea, what this is and
 how I
  handle this problem. Can anyone please help?
 
  Thanks a lot in advance!
  --
  View this message in context:
  http://www.nabble.com/Non-linear-regression-Quantile-regression-
 tp23944530p23944530.html
  Sent from the R help mailing list archive at Nabble.com.
 
  __
  R-help@r-project.org mailing list
  https://stat.ethz.ch/mailman/listinfo/r-help
  PLEASE do read the posting guide
  http://www.R-project.org/posting-guide.html
  and provide commented, minimal, self-contained, reproducible code.
 
 
  __
  R-help@r-project.org mailing list
  https://stat.ethz.ch/mailman/listinfo/r-help
  PLEASE do read the posting guide
  http://www.R-project.org/posting-guide.html
  and provide commented, minimal, self-contained, reproducible code.
 
 

 --
 View this message in context: http://www.nabble.com/Non-linear-
 regression-Quantile-regression-tp23944530p23945900.html
 Sent from the R help mailing list archive at Nabble.com.

 __
 R-help@r-project.org mailing list
 https://stat.ethz.ch/mailman/listinfo/r-help
 PLEASE do read the posting guide http://www.R-project.org/posting-
 guide.html
 and provide commented, minimal, self-contained, reproducible code.
 __
 R-help@r-project.org mailing list
 https://stat.ethz.ch/mailman/listinfo/r-help
 PLEASE do read the posting guide http://www.R-project.org/posting-guide.html
 and provide commented, minimal, self-contained, reproducible code.


__
R-help@r-project.org mailing list
https://stat.ethz.ch/mailman/listinfo/r-help
PLEASE do read the posting guide http://www.R-project.org/posting-guide.html
and provide commented, minimal, self-contained, reproducible code.


Re: [R] Non-linear regression with latent variable

2009-05-21 Thread spencerg
 If you have the RSiteSearch package installed, you can do the 
following: 



library(RSiteSearch)
nrow(nll - RSiteSearch.function(nonlinear regression with latent))
HTML(nll)


 This just produced 8 hits for me.  If this doesn't solve your 
problem, you might  try other search terms.  If all those fail, you 
could write your own likelihood function and use the maxLik package. 



 Hope this helps. 
 Spencer



Samiul Hasan wrote:
Hi 
Can anyone please suggest me a package where I can estimate a non-linear

regression model? One of the independent variables is latent or unobserved.
I have an indicator variable for this unobserved variable; however the
relationship is known to be non-linear also. In terms of equations my
problem is

y=f(latent, fixed) 
q=g(latent) where q is the indicator variable


For me both f and g are non-linear.

Thanks
Samiul Hasan


   



__
R-help@r-project.org mailing list
https://stat.ethz.ch/mailman/listinfo/r-help
PLEASE do read the posting guide http://www.R-project.org/posting-guide.html
and provide commented, minimal, self-contained, reproducible code.


Re: [R] non linear regression with nls

2009-02-06 Thread lauramorg...@bluewin.ch
Thank you! It worked perfectly, also for the other variables!

Messaggio originale
Da: r...@life.ku.dk
Data: 06.02.2009 13.29
A: lauramorg...@bluewin.ch
Oggetto: Re: [R] non linear regression with nls

Hi Laura,

I think you have to make a list formulas:


formList - list(NT.N ~ fz1(Portata, a, b), NT.N ~ fz2(Portata, a, b), NT.N ~ 
fz3(Portata,
a, b, d, e), NT.N ~ fz4(Portata, a, b), NT.N ~ fz5(Portata, a, b, d))


and then in the loop:

resultList[[i]] - nls(formList[[i]], ...



Christian

__
R-help@r-project.org mailing list
https://stat.ethz.ch/mailman/listinfo/r-help
PLEASE do read the posting guide http://www.R-project.org/posting-guide.html
and provide commented, minimal, self-contained, reproducible code.


Re: [R] non linear regression with nls

2009-02-05 Thread lauramorg...@bluewin.ch
Thank you a lot Mr. Ritz!
I've tried the loop you suggested and I added a list for lower and upper limits 
of the parameters, but there is still 
a problem... 

I have a list of functions, which works...
fz1-function(Portata, a, b){a+(b/Portata)} 
fz2-function(Portata, a, b){a*exp(b*Portata)}
fz3-function(Portata, a, b, d, e){a+(b/Portata)+d*(Portata^e)}
fz4-function(Portata, a, b){a*Portata^b}
fz5-function(Portata, a, b, d){a+b*(Portata^d)}
fctList - list(fz1, fz2, fz3, fz4, fz5)

as well as lists for starting values, upper and lower values, which work as 
well:
startList - list(list(a=10, b=10), list(a=10, b=1), list(a=10, b=10, d=10, 
e=1), list(a=10, b=1), list(a=10, b=10, 
d=1))
lowerList-list(list(a=0,b=0),list(a=0,b=0), list(a=0,b=0,d=0,e=-50),list(a=0, 
b=-50), list(a=0, b=0, d=-50))
upperList-list(list(a=1000, b=1000), list(a=1000, b=1000), 
list(a=1000,b=1000,d=1000,e=50), list(a=1000,b=50), list
(a=1000, b=1000, d=50))

but if I try to run this for loop


resultList - list()
for (i in 1:5)
{
resultList[[i]] - nls(NT.N ~ fctList[[i]](Portata, a,b), data=subset(dati, 
Fiume==Laveggio), start=startList[[i]], 
nls.control(maxiter=200), algorithm='port', trace=TRUE, na.action=na.omit, 
upper=upperList[[i]], lower=lowerList[[i]])
}
 I get the following error message:
Error in fctList[[i]](Portata, a, b) : element 1 is empty;
   the part of the args list of '*' being evaluated was:
   (d, (Portata^e))

I realized that the problem is the element after the function, i.e. (Portata, 
a, b), since fct 3 and 5 have more 
parameters (Portata,a,b,d,e).
So I tried to make a list (parList) for this too, i tried 2 versions:
#version1
list(Portata,a, b)-pf1.2.4
list(Portata,a,b,d,e)-pf3
list(Portata,a,b,d)-pf5
parList-list(pf1.2.4, pf1.2.4,pf3,pf1.2.4,pf5)

#version 2
parList-list(Portata,a,b,Portata,a,b,Portata,a,b,d,e, 
Portata,a,b,Portata,a,b,d)

and then I tried them (one at a time) in the loop:

resultList - list()
for (i in 1:5)
{
resultList[[i]] - nls(NT.N ~ fctList[[i]](parList[[i]]), data=subset(dati, 
Fiume==Laveggio), start=startList
[[i]], nls.control(maxiter=200), algorithm='port', trace=TRUE, 
na.action=na.omit,upper=upperList[[i]], lower=lowerList
[[i]])
}
but I got this error message:  Error in fctList[[i]](parList[i]) : element 1 
is empty;
   the part of the args list of '+' being evaluated was:
   (a, (b/Portata))

What can I do to fix it?
I'm also wondering which kind of function (maybe another loop?) I could use to 
automize the regression not only for 
the variable NT.N but for every variable (PTG.P, PO4.P,. ..)

My dataframe look like this (a sample):

Fiume giorno mese anno Portata  PTG.P   PO4.P   NT.N  NH4.N NO3.N   
BOD5 SiO2  data
1Vedeggio 101 1995   0.981 218.40 118.000  9.196 6.5700  2.06  
6.080 4.33 34709
2Vedeggio  72 1995   0.965 125.84  54.000  8.701 5.2600  2.31 
16.480 4.43 34737
3Vedeggio  73 1995   1.536  37.44  12.000  7.271 5.5600  1.88  
5.240 4.15 34765
...
190 Cassarate 299 2008   1.240  26.00  20.000  2.480 0.1200  1.79  
1.700 4.03 39720
191 Cassarate 13   10 2008   0.860  23.00  16.000  2.720 0.0200  2.13  
1.780 3.71 39734
192 Cassarate 10   11 2008   8.840  26.00  14.000  2.900 0.0500NA  
1.400 3.62 39762
193 Cassarate  9   12 2008   2.030  35.00  23.000  2.190 0.0700  1.79  
1.950 3.74 39791
...
279  Laveggio 151 2002   0.347  77.00  30.000  9.690 0.4300  7.23  
1.950 4.17 37271
280  Laveggio 112 2002   0.527  54.00  17.000  7.520 0.8800  5.87  
2.410 3.58 37298
281  Laveggio 133 2002   0.900  34.00  15.000  7.520 0.7100  6.17  
6.550 3.03 37328
...
Thanks to anyone that could give me any hint!!
Laura




## a for loop
resultList - list()
for (i in 1:5)
{
## storing the result as the i'th list component
## notice that the i'th list components in fctList and startList
## are used for the i'th fit
resultList[[i]] - nls(NT.N ~ fctList[[i]](parList[[i]]), data=subset(dati, 
Fiume==Laveggio), start=startList
[[i]], nls.control(maxiter=200), algorithm='port', trace=TRUE, 
na.action=na.omit)
}

Messaggio originale
Da: r...@life.ku.dk
Data: 03.02.2009 19.00
A: lauramorg...@bluewin.ch
Oggetto: Re: [R] non linear regression with nls

Hi Laura,

I've the following suggestion for you using several lists and a for loop:


fz1-function(Portata, a, b){a+b/Portata}
fz2-function(Portata, a, b){a*exp(b*Portata)}
fz3-function(Portata, a, b, d, e){a+b/Portata+d*(Portata^e)}
fz4-function(Portata, b, d){b*Portata^d}
fz5-function(Portata, a, b, d){a+b*(Portata^d)}
fctList - list(fz1, fz2, fz3, fz4, fz5)

startList - list(list(a=10, b=10), list(a=10, b=1), start=list(a=10, b=10, 
d=10, e=10),
list(a=10, b=1), list(a=10, b=10, d=1))

## a for loop
resultList - list()
for (i in 1:5)
{
## storing the result as the i'th list component
## notice that the i'th list components in fctList and startList
## are used for the i'th fit
resultList[[i]] - nls(NT.N

Re: [R] non linear regression with nls

2009-02-05 Thread lauramorg...@bluewin.ch
Hello, thanks for the advice of nlsList!
I tried to look at the help page of nlsList, but I didnt understand how to use 
the subset argument of the function 
and it's not clear to 
me if this only allows you to choose one subset or if it run the regression for 
every given subset, in this case how 
can someone specify the groups/subset?
Thanks a lot!
Laura


Messaggio originale
Da: kfr...@wisc.edu
Data: 03.02.2009 15.36
A: lauramorg...@bluewin.ch
Oggetto: Re: [R] non linear regression with nls

Hi, Laura-

You might have a look at ?nlsList().

Ken


- Original Message -
From: lauramorg...@bluewin.ch lauramorg...@bluewin.ch
Date: Tuesday, February 3, 2009 4:38 am
Subject: [R] non linear regression with nls
To: r-help@r-project.org


 Hello,
 I'm a beginner with R and it's the first time I'm using the R-help 
 list... I hope I'm in the right place, if not: 
 Sorry!!
 
 I need to do non linear regressions on a data set which columns are:
 river.namePortata  PTG.P   PO4.P   NT.NNH4.N   
 NO3.N   BOD5SiO2   
 I need to predict every variable (PTG, PO4, NT, ..., which are 
 concentration of substances in water) starting from 
 the Portata variable (which is the water flow)
 The functions that I'm using are:
  fz1-function(Portata, a, b){a+b/Portata}
  fz2-function(Portata, a, b){a*exp(b*Portata)}
  fz3-function(Portata, a, b, d, e){a+b/Portata+d*(Portata^e)}
  fz4-function(Portata, b, d){b*Portata^d}
  fz5-function(Portata, a, b, d){a+b*(Portata^d)}
 I've made a list of the functions with list(fz1, fz2, fz3, fz4, fz5)
 
 and the starting , lower and upper parameters for each function are:
 fz1: start=list(a=10, b=10), lower=list(a=0, b=0), upper=list(a=1000, 
 b=1000) 
 fz2: start=list(a=10, b=1), lower=list(a=0, b=0), upper=list(a=1000, b=1000)
 fz3: start=list(a=10, b=10, d=10, e=10), lower=list(a=0, b=0, d=0, 
 e=-50), upper=list(a=1000, b=1000, d=1000, e=50) 
 fz4: start=list (a=10, b=1), lower=list(a=0, b=-50), 
 upper=list(a=1000, b=50)
 fz5: start=list(a=10, b=10, d=1), lower=list(a=0, b=0, d=-50), 
 upper=list(a=1000, b=1000, d=50)
 
 so far i manage to do non linear regression one at a time that is, 
 using one function for one river) using nls(), for 
 example:
 r.NT.lav-nls(NT.N~ fz1(Portata, a,b), 
 data=subset(dati,river.name==Laveggio), start=list(a=10, b=10), nls.control
 (maxiter=200), algorithm='port', trace=TRUE, na.action=na.omit, 
 lower=list(a=0, b=0), upper=list(a=1000, b=1000))
 and then I get the results with summary() and the graph using plotfit()
 
 but this will get extremly long since I have 12 rivers to analize for 
 every variable and then compare the results, so 
 I'd like to use a loop (cycle for??) but I can't figure out how it 
 works. I've tried to look on the help page on ?
 Control (control flow) but I didn't understand it... 
 Can somebody help me (give me a hint or an example of a loop) to 
 automize the regression and save the results
 Please consider that my knowledge of computer programming is 
 practically non-existent!!
 Thanks a lot!
 Laura  F.
 
 __
 R-help@r-project.org mailing list
 https://stat.ethz.ch/mailman/listinfo/r-help
 PLEASE do read the posting guide http://www.R-project.org/posting-guide.html
 and provide commented, minimal, self-contained, reproducible code.

__
R-help@r-project.org mailing list
https://stat.ethz.ch/mailman/listinfo/r-help
PLEASE do read the posting guide http://www.R-project.org/posting-guide.html
and provide commented, minimal, self-contained, reproducible code.


Re: [R] Non linear regression with 2 explanatory variables

2008-01-16 Thread Gavin Simpson
hits=-2.6 tests=BAYES_00
X-USF-Spam-Flag: NO

On Wed, 2008-01-16 at 11:02 +0100, Janice Kielbassa wrote:
 Hello!
 
 I want to do a non-linear regression with 2 explanatory variables 
 (something like : length ~ a * time * exp( b* temperature)), having a 
 data set (length, time, temperature). Which function could I use (I 
 tried nls but I think it doesn't work)

Janice, I'll start by saying I can't help you as I have never used nls()
myself and I am not familiar with this type of analysis.

Why do you think that nls() doesn't work? It is a widely used part of
R and thus probably very well tested.

My understanding of these things is that nls is a sophisticated tool
that requires some effort on the part of the user, such as selecting
appropriate starting values.

You are unlikely to get any further assistance from the list unless you
read the posting guide and post an example of what you did (preferably
with the actual data or dummy data with the same properties if not) and
the exact error message or output from R that lead you to believe that
nls() did not work.

HTH

G

 Thanks a lot!
 
 Janice
 
 __
 R-help@r-project.org mailing list
 https://stat.ethz.ch/mailman/listinfo/r-help
 PLEASE do read the posting guide http://www.R-project.org/posting-guide.html
 and provide commented, minimal, self-contained, reproducible code.
-- 
%~%~%~%~%~%~%~%~%~%~%~%~%~%~%~%~%~%~%~%~%~%~%~%~%~%~%~%~%~%~%~%~%~%~%
 Dr. Gavin Simpson [t] +44 (0)20 7679 0522
 ECRC, UCL Geography,  [f] +44 (0)20 7679 0565
 Pearson Building, [e] gavin.simpsonATNOSPAMucl.ac.uk
 Gower Street, London  [w] http://www.ucl.ac.uk/~ucfagls/
 UK. WC1E 6BT. [w] http://www.freshwaters.org.uk
%~%~%~%~%~%~%~%~%~%~%~%~%~%~%~%~%~%~%~%~%~%~%~%~%~%~%~%~%~%~%~%~%~%~%

__
R-help@r-project.org mailing list
https://stat.ethz.ch/mailman/listinfo/r-help
PLEASE do read the posting guide http://www.R-project.org/posting-guide.html
and provide commented, minimal, self-contained, reproducible code.


Re: [R] Non linear regression with 2 explanatory variables

2008-01-16 Thread Roland Rau
Gavin Simpson wrote:
 hits=-2.6 tests=BAYES_00
 X-USF-Spam-Flag: NO
 
 On Wed, 2008-01-16 at 11:02 +0100, Janice Kielbassa wrote:
 Hello!

 I want to do a non-linear regression with 2 explanatory variables 
 (something like : length ~ a * time * exp( b* temperature)), having a 
 data set (length, time, temperature). Which function could I use (I 
 tried nls but I think it doesn't work)
 
 Janice, I'll start by saying I can't help you as I have never used nls()
 myself and I am not familiar with this type of analysis.
 

maybe it helps if you have a look at Chapter 10 Nonlinear Models by 
Douglas M. Bates and John M. Chambers in: John M. Chambers, Trevor J. 
Hastie (Eds.): Statistical Models in S. Chapman  Hall/CRC , 1992

Best,
Roland

__
R-help@r-project.org mailing list
https://stat.ethz.ch/mailman/listinfo/r-help
PLEASE do read the posting guide http://www.R-project.org/posting-guide.html
and provide commented, minimal, self-contained, reproducible code.


Re: [R] Non linear regression with 2 explanatory variables

2008-01-16 Thread Rolf Turner

I have never had much success in using nls().  If you scan the archives
you will find one or two postings from me on this topic.  I have  
received
no useful responses to these postings.

I have found that anything that I tried (and failed) to do using nls()
could be done quite easily using optim().

cheers,

Rolf Turner


On 17/01/2008, at 3:56 AM, Gavin Simpson wrote:

 hits=-2.6 tests=BAYES_00
 X-USF-Spam-Flag: NO

 On Wed, 2008-01-16 at 11:02 +0100, Janice Kielbassa wrote:
 Hello!

 I want to do a non-linear regression with 2 explanatory variables
 (something like : length ~ a * time * exp( b* temperature)), having a
 data set (length, time, temperature). Which function could I use (I
 tried nls but I think it doesn't work)

 Janice, I'll start by saying I can't help you as I have never used  
 nls()
 myself and I am not familiar with this type of analysis.

 Why do you think that nls() doesn't work? It is a widely used  
 part of
 R and thus probably very well tested.

 My understanding of these things is that nls is a sophisticated tool
 that requires some effort on the part of the user, such as selecting
 appropriate starting values.


##
Attention:\ This e-mail message is privileged and confid...{{dropped:9}}

__
R-help@r-project.org mailing list
https://stat.ethz.ch/mailman/listinfo/r-help
PLEASE do read the posting guide http://www.R-project.org/posting-guide.html
and provide commented, minimal, self-contained, reproducible code.


Re: [R] Non linear regression with 2 explanatory variables

2008-01-16 Thread Katharine Mullen
Dear Rolf,

One thing that sometimes makes nls easier to apply is using the 'formula'
argument like you would use the 'fn' argument of optim.  That is, if you
have a residual function that has arguments x, y, a, b and you need to
optimize a and b, you would make a call like

nls(~resid(x,y,a=astart, b=bstart), control = nls.control(warnOnly =
   TRUE, printEval = TRUE), start = list(a=astart, b=bstart))

This did not work easily before R-2.6.0, but does now.  The Puromycin
analysis from the help files is an example of this useage and below is
another.

Or do you already use nls this way and still have problems?

# get data as a sum of exponentials
dataSumOfExp - function(rates = seq(.05, .005, length=3),
 times = 1:100,
 amps = rep(1, length(rates))) {
  tfun - function(t,r) exp(-r*t)
  ## get C with tfun
  C - mapply(tfun, r=rates, MoreArgs=list(t=times))
  ## add the columns of C with relative amplitudes 1, and add noise
  C %*% amps + rnorm( nrow(C) )  * max(C) * .1
}

# residual function
resFun - function(rates, amps, measured, times = 1:100) {
  tfun - function(t,r) exp(-r*t)
  CEst - mapply(tfun, r=rates, MoreArgs=list(t=times))
  measured - CEst %*% amps
}

# get data
measured - dataSumOfExp()

# optimize rates of exponentials and their relative amplitudes
res - nls(~resFun(rates = rates, measured = measured, amps = amps),
   control = nls.control(warnOnly = TRUE, printEval = TRUE),
   start = list(rates = c(.04, .1, .001),
 amps = rep(1,3)), trace = TRUE)

summary(res)

On Thu, 17 Jan 2008, Rolf Turner wrote:


 I have never had much success in using nls().  If you scan the archives
 you will find one or two postings from me on this topic.  I have
 received
 no useful responses to these postings.

 I have found that anything that I tried (and failed) to do using nls()
 could be done quite easily using optim().

   cheers,

   Rolf Turner


 On 17/01/2008, at 3:56 AM, Gavin Simpson wrote:

  hits=-2.6 tests=BAYES_00
  X-USF-Spam-Flag: NO
 
  On Wed, 2008-01-16 at 11:02 +0100, Janice Kielbassa wrote:
  Hello!
 
  I want to do a non-linear regression with 2 explanatory variables
  (something like : length ~ a * time * exp( b* temperature)), having a
  data set (length, time, temperature). Which function could I use (I
  tried nls but I think it doesn't work)
 
  Janice, I'll start by saying I can't help you as I have never used
  nls()
  myself and I am not familiar with this type of analysis.
 
  Why do you think that nls() doesn't work? It is a widely used
  part of
  R and thus probably very well tested.
 
  My understanding of these things is that nls is a sophisticated tool
  that requires some effort on the part of the user, such as selecting
  appropriate starting values.


 ##
 Attention:\ This e-mail message is privileged and confid...{{dropped:9}}

 __
 R-help@r-project.org mailing list
 https://stat.ethz.ch/mailman/listinfo/r-help
 PLEASE do read the posting guide http://www.R-project.org/posting-guide.html
 and provide commented, minimal, self-contained, reproducible code.


__
R-help@r-project.org mailing list
https://stat.ethz.ch/mailman/listinfo/r-help
PLEASE do read the posting guide http://www.R-project.org/posting-guide.html
and provide commented, minimal, self-contained, reproducible code.